Naked Science Forum

On the Lighter Side => New Theories => Topic started by: hamdani yusuf on 08/12/2020 13:15:09

Title: Where does quantization of energy of electromagnetic radiation come from?
Post by: hamdani yusuf on 08/12/2020 13:15:09
According to Planck's Law, energy of radiation is quantized.
E = n.h.f
n is integer.
h is Planck's constant
f is frequency

Dimensional analysis tells us that energy has mass and length in it. In the equation above, they reside in Planck's constant. So the quantification of energy must come from some things that determine the value of Planck's constant.

We have learned that electromagnetic radiation comes from moving electrically charged particles. They have kinetic energy according to the formula
Ek=½.m.v²
m is the mass and v is the speed
The particles also have potential energy which depends on their electric charge.
(https://upload.wikimedia.org/wikipedia/commons/f/f6/Electric_potential_energy_3_charge.gif)
Electrostatic potential energy of q due to Q1 and Q2 charge system:
(https://wikimedia.org/api/rest_v1/media/math/render/svg/248eae80607ba9e9ec3cee3760dd2ea7d8a5b64b)
https://en.wikipedia.org/wiki/Electric_potential_energy#One_point_charge_q_in_the_presence_of_n_point_charges_Qi
Those particles have discrete values of mass and electric charge. So it comes naturally that their electromagnetic radiation would come in quantified amounts.
Title: Re: Where does quantization of energy of electromagnetic radiation come from?
Post by: puppypower on 09/12/2020 14:40:01
If you compare quantized energy levels, to the hypothetical energy levels that would be expected using continuous mathematical functions, quantized places a narrower limit on what is possible. Not all states are possible in the quantum world.

 From a practical POV, quantum saves time; lowers time potential. In other words, if we needed a given value to appear, so we can go from step 1 to step 2, by placing a quantum restriction, we will reach the needed state much sooner, throwing dice, compared to if all all states were continuous and possible. This allows the universe to evolve faster; saves time or speeds up evolution.

The affect is similar to loading dice. If we place a load on a dice, the odds will increase for certain outcomes, based on where the load has been placed. Other outcomes become unlikely, since they are in the gaps between the quanta This makes a purely random universe more predicable and deterministic; betting odds improve.
Title: Re: Where does quantization of energy of electromagnetic radiation come from?
Post by: Bored chemist on 09/12/2020 17:41:55
Those particles have discrete values of mass and electric charge. So it comes naturally that their electromagnetic radiation would come in quantified amounts.
Not really.
In classical physics, there is nothing to stop r1 or r2 taking any value it likes.
Title: Re: Where does quantization of energy of electromagnetic radiation come from?
Post by: hamdani yusuf on 10/12/2020 03:38:44
Not really.
In classical physics, there is nothing to stop r1 or r2 taking any value it likes
In a stable orbital motion, the radius depends on centripetal force. In electromagnetic interaction, it depends on electric charges of the particles involved. Since the charges can only change in multiplication of integer, so does the radius.
Title: Re: Where does quantization of energy of electromagnetic radiation come from?
Post by: evan_au on 10/12/2020 06:57:33
Quote from: hamdani yusuf
Since the charges can only change in multiplication of integer, so does the radius.
For a given atom, the electron can exist in one of a potentially infinite number of orbitals.
- The charge on the electron is always -1, regardless of which orbital it is in
- The charge on the nucleus is fixed and positive
- The product of electron charge and nucleus charge is fixed

So how come you can have several different permitted radii, if the product of the charges is fixed?
- That's not it!

Quote
In a stable orbital motion, the radius depends on centripetal force.
When a planet is orbiting the Sun in a circular orbit, the attraction between the Sun and planet balances the tendency of the planet to fly off into space in a straight line (hence your comment that "In a stable orbital motion, the radius depends on centripetal force").
- There are an infinite number of combinations of radius and velocity where these forces balance for a circular orbit, so this is not the source of quantization.
- It gets more complicated if you try to account for elliptical orbits of planets, as the gravitational force does not balance the centripetal force for most of the orbit.
- And most planets have elliptical orbits

In atoms, only certain orbitals are permitted (quantization)
- To calculate these orbitals, you need to solve the wave equation for the electron.
- Some of these orbitals are spherical, but others look like a cluster of balloons assembled by a clown. How do you calculate the radius and centripetal motion for these?

If you want a simple understanding, have a look at Bohr's model of the atom, where an electron's angular momentum is quantized (classical physics has no equivalent).
- Or de Broglie's model where the electron has a wavelength, and that wavelength must have an integer number of wavelengths to be stable (classical physics has no equivalent).
- But for a good model, you have to solve the relativistic Schroedinger equation, which gets quite complex for anything bigger than a hydrogen atom. Even a Hydrogen atom is beyond what they are paying me here!

See: https://en.wikipedia.org/wiki/Atomic_orbital#Bohr_atom
Title: Re: Where does quantization of energy of electromagnetic radiation come from?
Post by: Bored chemist on 10/12/2020 08:55:35
In a stable orbital motion,
In classical physics, any orbital radius is allowed- as long as the velocity matches it.
Title: Re: Where does quantization of energy of electromagnetic radiation come from?
Post by: hamdani yusuf on 10/12/2020 10:13:34
F centripetal = F coulomb
m.v²/r = k.q1.q2/r²
m.ω².r = k.q1.q2/r²
ω².r³ = k.q1.q2/m
The value in the right side is quantized, but the left side is not. But the equation above only take electrostatic force into account. The additional force due to electrodynamic (magnetic) hasn't been included. I guess that the formula for electrodynamic force can be set to make the radiation energy per frequency quantized.
Title: Re: Where does quantization of energy of electromagnetic radiation come from?
Post by: Bored chemist on 10/12/2020 11:43:21
Neither side "needs" to be quantised.
the radius is a continuous variable.
Title: Re: Where does quantization of energy of electromagnetic radiation come from?
Post by: hamdani yusuf on 11/12/2020 05:01:50
Quote from: hamdani yusuf
Since the charges can only change in multiplication of integer, so does the radius.
For a given atom, the electron can exist in one of a potentially infinite number of orbitals.
- The charge on the electron is always -1, regardless of which orbital it is in
- The charge on the nucleus is fixed and positive
- The product of electron charge and nucleus charge is fixed

So how come you can have several different permitted radii, if the product of the charges is fixed?
- That's not it!

Quote
In a stable orbital motion, the radius depends on centripetal force.
When a planet is orbiting the Sun in a circular orbit, the attraction between the Sun and planet balances the tendency of the planet to fly off into space in a straight line (hence your comment that "In a stable orbital motion, the radius depends on centripetal force").
- There are an infinite number of combinations of radius and velocity where these forces balance for a circular orbit, so this is not the source of quantization.
- It gets more complicated if you try to account for elliptical orbits of planets, as the gravitational force does not balance the centripetal force for most of the orbit.
- And most planets have elliptical orbits

In atoms, only certain orbitals are permitted (quantization)
- To calculate these orbitals, you need to solve the wave equation for the electron.
- Some of these orbitals are spherical, but others look like a cluster of balloons assembled by a clown. How do you calculate the radius and centripetal motion for these?

If you want a simple understanding, have a look at Bohr's model of the atom, where an electron's angular momentum is quantized (classical physics has no equivalent).
- Or de Broglie's model where the electron has a wavelength, and that wavelength must have an integer number of wavelengths to be stable (classical physics has no equivalent).
- But for a good model, you have to solve the relativistic Schroedinger equation, which gets quite complex for anything bigger than a hydrogen atom. Even a Hydrogen atom is beyond what they are paying me here!

See: https://en.wikipedia.org/wiki/Atomic_orbital#Bohr_atom
I want to know how far we can follow classical physics until it inevitably fails and no reasonable assumptions can be put into it to make it work and agree with observations.
Title: Re: Where does quantization of energy of electromagnetic radiation come from?
Post by: hamdani yusuf on 11/12/2020 05:23:31
Neither side "needs" to be quantised.
the radius is a continuous variable.
If we accept that mass and charges are quantized, while k is constant, then it is necessary that value of (k. q. q/m) is quantized.
Planck's law only suggests for quantized radiation energy, while the frequency itself is not quantized so it can have any value.
Title: Re: Where does quantization of energy of electromagnetic radiation come from?
Post by: evan_au on 11/12/2020 07:26:28
Quote from: Yusuf Hamdani
ω².r³ = k.q1.q2/m
I agree that everything on the right side is quantized.

So that leaves an infinite number of combinations of ω & r on the left hand side, ie neither ω nor r are quantized by this equation.

Quote
I want to know how far we can follow classical physics until it inevitably fails
Classical physics, with no quantisation of orbital energy immediately collapses in an "ultraviolet catastrophe".
I have seen suggestions that all atoms would collapse into neutrons within femtoseconds!
The Earth would collapse into a neutron planet within 15 minutes.

See: https://en.wikipedia.org/wiki/Ultraviolet_catastrophe
Title: Re: Where does quantization of energy of electromagnetic radiation come from?
Post by: hamdani yusuf on 11/12/2020 09:12:22
Classical physics, with no quantisation of orbital energy immediately collapses in an "ultraviolet catastrophe".
There are many versions of classical physics, other than Newtonian and Maxwellian theories. Ultraviolet catastrophe is not the only failure of Maxwellian physics.
I want to know what's the cause of that quantization in a more fundamental level,  rather than taking it as a postulate.
Title: Re: Where does quantization of energy of electromagnetic radiation come from?
Post by: hamdani yusuf on 14/10/2021 07:17:58
A Simple Method For Measuring Plancks Constant
Quote
The discovery of Planck's constant in the year 1900 was one of the most important discoveries that catalyzed the quantum revolution. What started as a simple idea to resolve one of the greatest physics mysteries of the time, turned out to be the key to unlocking the quantum realm. While Planck assumed that the constant would be 0 when measured the constant had a definite and real value, meaning that there was a lower limit on the universe.  Preforming a basic version of this measurement is actually really easy and we explore the process of that measurement in this video using some LEDs and a diffraction grating.
Title: Re: Where does quantization of energy of electromagnetic radiation come from?
Post by: Bored chemist on 15/10/2021 18:19:15
A Simple Method For Measuring Plancks Constant
Quote
The discovery of Planck's constant in the year 1900 was one of the most important discoveries that catalyzed the quantum revolution. What started as a simple idea to resolve one of the greatest physics mysteries of the time, turned out to be the key to unlocking the quantum realm. While Planck assumed that the constant would be 0 when measured the constant had a definite and real value, meaning that there was a lower limit on the universe.  Preforming a basic version of this measurement is actually really easy and we explore the process of that measurement in this video using some LEDs and a diffraction grating.
Interesting, but more or less wrong.
The brightness vs voltage curve for an LED is a smooth function, quite close to an exponential.
So there's no voltage where the light suddenly "turns on".
Here are some examples. (The light output is proportional to the current).
https://www.researchgate.net/figure/Current-versus-voltage-I-V-curves-for-pixels-of-differing-diameter-from-8-8-micro-LED_fig1_260327577
Title: Re: Where does quantization of energy of electromagnetic radiation come from?
Post by: hamdani yusuf on 16/10/2021 05:09:36
Here's the diagram from the research mentioned in BC's post.
(https://www.researchgate.net/profile/Shuailong-Zhang/publication/260327577/figure/fig1/AS:392480626364429@1470586081886/Current-versus-voltage-I-V-curves-for-pixels-of-differing-diameter-from-8-8-micro-LED.png)

Quote
We report the high-frequency modulation of individual pixels in 8 × 8 arrays of III-nitride-based micro-pixellated light-emitting diodes, where the pixels within the array range from 14 to 84 μ m in diameter. The peak emission wavelengths of the devices are 370, 405, 450 and 520 nm, respectively.
It seems like the experiment doesn't explicitly support the quantization of energy.
Title: Re: Where does quantization of energy of electromagnetic radiation come from?
Post by: hamdani yusuf on 16/10/2021 05:49:46
Imagine an electron oscillating up and down in 1 Hz frequency. What's the amplitude of the oscillation which is corresponding to minimum energy transfer, which is h? Why can't we get lower than that while still non zero?
Title: Re: Where does quantization of energy of electromagnetic radiation come from?
Post by: hamdani yusuf on 10/05/2023 16:53:11
Dimensional analysis tells us that energy has mass and length in it. In the equation above, they reside in Planck's constant. So the quantification of energy must come from some things that determine the value of Planck's constant.
Let's analyze Planck's formula by plotting it in a graphic. To avoid unnecessary complications, the units are chosen so that every constant has value of 1.
https://www.wolframalpha.com/input?i=plot++1+f%5E3%2F%28e%5E%281+f%29+-1%29+from+0+to+20

What will happen if the Planck's constant were smaller or bigger than that?

https://www.wolframalpha.com/input?i=plot++0.5+f%5E3%2F%28e%5E%280.5+f%29+-1%29+from+0+to+20

https://www.wolframalpha.com/input?i=plot++2+f%5E3%2F%28e%5E%282+f%29+-1%29+from+0+to+10

It seems that the curve maintains it's shape, and only the scales in x and y axes change. It means that quantization of radiation energy can't be derived solely from Planck's formula. It must come from somewhere else.
Title: Re: Where does quantization of energy of electromagnetic radiation come from?
Post by: hamdani yusuf on 11/05/2023 13:00:01
Imagine an electron oscillating up and down in 1 Hz frequency. What's the amplitude of the oscillation which is corresponding to minimum energy transfer, which is h? Why can't we get lower than that while still non zero?
Let's approach the problem from different direction, while keeping it simple.
Imagine an electron oscillating up and down in 1 Hz frequency, and the amplitude of the oscillation is 1 meter. Is there any way to determine its radiation power?
Title: Re: Where does quantization of energy of electromagnetic radiation come from?
Post by: hamdani yusuf on 11/05/2023 13:24:56
SI unit analysis tells us that h is stated in Joule second per cycle. The cycle is often omitted because it has no dimension. But IMO, this can lead to confusions. Why can't we say that the unit is Joule second per cycle square?

According to current quantum theories, the minimum non-zero energy can be radiated by an antenna transmitting 1 Hz electromagnetic wave is h Joule. But there's no minimum power, hence the power output can be made arbitrarily close to zero, given an extremely long period to radiate it.
Title: Re: Where does quantization of energy of electromagnetic radiation come from?
Post by: hamdani yusuf on 27/05/2023 16:34:35
This video tries to find analogy for quantification of electromagnetic radiation frequencies in atomic spectrum in wave physical framework. But it doesn't explicitly state why the energy for a certain frequency is quantized.
Quote
Trying to find analogies between the wave energy confined in a string and matter interacting with light.

0:00 Intro
6:38 Experiments with waves in a string
15:40 Analogies with electron waves
22:50 Changing the standing wave mode in a string using phase
26:49 A hypothetical model for demonstrating quantized wave behavior
32:26 Elastically Inertial Poetry

When I learned about Balmer formula, I also imagined about beat frequency. This video shows how to visualize it.
Title: Re: Where does quantization of energy of electromagnetic radiation come from?
Post by: hamdani yusuf on 28/05/2023 09:17:42
Quote
https://en.m.wikipedia.org/wiki/Rydberg_formula
As stressed by Niels Bohr,[2] expressing results in terms of wavenumber, not wavelength, was the key to Rydberg's discovery. The fundamental role of wavenumbers was also emphasized by the Rydberg-Ritz combination principle of 1908. The fundamental reason for this lies in quantum mechanics. Light's wavenumber is proportional to frequency
1/L=f/c, and therefore also proportional to light's quantum energy E. Thus,
1/L=E/hc (in this formula the h represents Planck's constant).
It can be interpreted that the amount of radiation energy is proportional to the number of waves, regardless of how much time is elapsed to radiate it. In effect, given the same number of oscillators, low frequency radiation took longer to radiate a specified amount of energy, compared to higher frequency.
Title: Re: Where does quantization of energy of electromagnetic radiation come from?
Post by: alancalverd on 28/05/2023 15:19:47
So there's no voltage where the light suddenly "turns on".
Oh yes there is!

The I/V curve isn't definitive - the transiting electron must have sufficient energy to generate a photon of the appropriate color, so the emitted spectrum is related to the critical forward voltage of the diode. Vf increases with increasing current because the semiconductor process has a resistive component, but any particular LED behaves as a "conventional" (non-light-emitting) diode below Vfcrit. Above that value the intensity of emission increases with current, usually limited by heating which reduces efficiency, but the spectrum is relatively fixed.
Title: Re: Where does quantization of energy of electromagnetic radiation come from?
Post by: Bored chemist on 28/05/2023 16:16:59
Oh yes there is!

No, there's not.
Thermal energy can make a contribution to promoting an electron into an excited state.  (which is also why the V/I curve is temperature dependent.)

What voltage do you calculate that you need to get, for example, a green LED to light up?
Something like this...
https://uk.farnell.com/multicomp/mcl053gd/led-5mm-36-green/dp/1581138?
Title: Re: Where does quantization of energy of electromagnetic radiation come from?
Post by: alancalverd on 29/05/2023 14:29:46
If you look at the pdf data sheet you will see that there is "leakage only" current below about 1.65 V, and (obviously) no light emission at zero junction current. Since the lowest visible energy is about 1.2 eV, conservation laws suggest that even if we add a bit of thermal energy, you need at least a volt from somewhere to get any visible light from an LED. UV LEDs typically run at 4 - 8 volt Vf.
Title: Re: Where does quantization of energy of electromagnetic radiation come from?
Post by: alancalverd on 29/05/2023 14:39:03
Returning to the original question, EMR is not necessarily quantised. Maxwell's propagation laws apply at any and all frequencies, so you can in principle generate radiation at any photon energy you want.

What Planck said was IF you have an ideal particle rattling about in a perfectly elastic box, it can only have discrete energy levels, and you can use this model to predict the UV spectrum etc. If the box does not have defined dimensions then the number of permissible states tends to infinity, hence the black body continuum with a continuous distribution of energy versus wavelength as predicted by the rigid box model for any particular wavelength.
Title: Re: Where does quantization of energy of electromagnetic radiation come from?
Post by: hamdani yusuf on 29/05/2023 14:52:40
Above that value the intensity of emission increases with current, usually limited by heating which reduces efficiency, but the spectrum is relatively fixed.
The video shows that the spectrum is temperature dependent.
Title: Re: Where does quantization of energy of electromagnetic radiation come from?
Post by: hamdani yusuf on 29/05/2023 15:12:23
Returning to the original question, EMR is not necessarily quantised. Maxwell's propagation laws apply at any and all frequencies, so you can in principle generate radiation at any photon energy you want.
Afaik, Maxwell's model for electromagnetic radiation has no concept of photon.
Title: Re: Where does quantization of energy of electromagnetic radiation come from?
Post by: Bored chemist on 29/05/2023 16:30:18
Since the lowest visible energy is about 1.2 eV...
If you can see 1330nm.
For human, a better estimate is about 1.75eV  Or 708 nm.
So, no LED should produce visible light if driven with less than 1.75V; is that right?

And, if you take the "limit" down to 1.5V you get a cut-off wavelength of 826 nm- which is invisible.
If you are right, a single "1.5 v" dry cell couldn't produce visible light from an LED even if you allow that a new one might produce about 1.6V...

Regular viewers may be able to work out what happens next.
:-)
Title: Re: Where does quantization of energy of electromagnetic radiation come from?
Post by: alancalverd on 29/05/2023 19:21:18
Afaik, Maxwell's model for electromagnetic radiation has no concept of photon.
But ours does. And Maxwell doesn't have a model - he derived a selfpropagating wave from the mathematics of known experimental electromagnetic phenomena.
Title: Re: Where does quantization of energy of electromagnetic radiation come from?
Post by: alancalverd on 29/05/2023 19:27:48
OK, BC, I'm intrigued. Where does the photon energy come from if the device has no internal potential differences exceeding 1.5V?
Title: Re: Where does quantization of energy of electromagnetic radiation come from?
Post by: Bored chemist on 29/05/2023 20:16:21
Where does the photon energy come from
It's still  because of this.
Thermal energy can make a contribution to promoting an electron into an excited state.  (which is also why the V/I curve is temperature dependent.)
This is sort-of related.
https://www.esa.int/gsp/ACT/projects/electroluminescent_cooling_using_LEDs/
Title: Re: Where does quantization of energy of electromagnetic radiation come from?
Post by: alancalverd on 29/05/2023 22:36:57
I haven't found a credible estimate of the thermal energy of a semiconductor electron at room temperature. Classically it would be around 0.03 eV, scarcely significant, but one authoritative source calculates 5 eV, so the LED would be generating ultraviolet radiation spontaneously,which seems a bit unlikely too.
Title: Re: Where does quantization of energy of electromagnetic radiation come from?
Post by: hamdani yusuf on 29/05/2023 23:03:49
Afaik, Maxwell's model for electromagnetic radiation has no concept of photon.
But ours does. And Maxwell doesn't have a model - he derived a selfpropagating wave from the mathematics of known experimental electromagnetic phenomena.
He built a mathematical model based on wave mechanics. In his model, light is propagating electromagnetic wave in a medium.
Title: Re: Where does quantization of energy of electromagnetic radiation come from?
Post by: Bored chemist on 30/05/2023 09:06:47
one authoritative source calculates 5 eV
Define "authoritative"...
Title: Re: Where does quantization of energy of electromagnetic radiation come from?
Post by: alancalverd on 30/05/2023 12:36:21
University of Michigan physics department lecture notes.
Title: Re: Where does quantization of energy of electromagnetic radiation come from?
Post by: alancalverd on 30/05/2023 12:39:34
In his model, light is propagating electromagnetic wave in a medium.
No medium was required or specified. μ0 and ε0 are arbitrary constants that relate observation to common units of measurement.
Title: Re: Where does quantization of energy of electromagnetic radiation come from?
Post by: Bored chemist on 30/05/2023 13:13:02
Not very plausible because 5eV is enough to tear apart most molecules.
https://en.wikipedia.org/wiki/Bond-dissociation_energy#Representative_bond_enthalpies

conservation laws suggest
Your appeal to the conservation laws is good, but you are applying them the wrong way round.

Because the band gap corresponds to enough energy to produce a visible photon, it figures that ant electron crossing that gap causes the emission of one.
OK, in reality, the efficiency isn't 100% but the electrons falling down don't have much idea what other electrons are doing, so the yield is pretty nearly independent of current.
The light output is pretty much proportional to current.
So you need to look at the variation of current with voltage.
That's temperature dependent because an electron that has nearly enough energy to get over the barrier can be promoted thermally over it.
https://en.wikipedia.org/wiki/Shockley_diode_equation



Title: Re: Where does quantization of energy of electromagnetic radiation come from?
Post by: Eternal Student on 30/05/2023 16:12:29
Hi.

    I've always thought that Quantisation is fairly odd and can only be used with caution.  Although we can use Quantum mechanics to establish that quantisation should be there,  we then quickly establish that we cannot observe a perfect example of it.   Instead what we will typically observe is a frequency that could fall anywhere within some continuous range, with just some statements we can make about the probability distribution or spread of what is typically a continuous random variable.

     An atom with an electron in an excited state, should eventually have that electron fall back to a lower energy orbit but we don't really know exactly when that will happen.  Although the transition from one orbit to another should be quantised, there is an uncertainty relation between the time the atom takes to transition (remains in the excited state before falling back to the lower energy state) and the energy of the photon released.
    See  https://phys.libretexts.org/Bookshelves/University_Physics/Book%3A_University_Physics_(OpenStax)/University_Physics_III_-_Optics_and_Modern_Physics_(OpenStax)/07%3A_Quantum_Mechanics/7.03%3A_The_Heisenberg_Uncertainty_Principle
   for some discussion  (especially the section around equation 7.3.2 and example 7.3.3).

   As a consequence, all atomic spectral emission / absorption lines have some non-zero width rather than being perfect spikes at precisely one frequency.   There is another theoretical limitation that can be considered:  We also have a position - momentum uncertainty relation.   The atom which emitted the photon along with the detector that captured and identified it can be moving relative to the lab frame and a Doppler-shift in frequency is inevitable.   
    In practice there are also limitations on the accuracy of the equipment and random experimental errors that appear.   Even if you overlook those experimental limitations, the theoretical limitations from uncertainty relations cannot be avoided.   Overall, there should be some precise quantisation BUT you can't observe it in any single measurement.   Theoretically, we can only assert that the expected frequency of a photon emitted by an excited atom should correspond to the difference in the energy between the two states of the atom.   There is nothing we can do to remove all of the randomness and spread of the actual frequency that is really detected from any one atom and one emission.

Best Wishes.
Title: Re: Where does quantization of energy of electromagnetic radiation come from?
Post by: paul cotter on 30/05/2023 20:44:43
Hi, ES. This is not my area but i'll hazard a guess with respect to the width of emission spectra. There must be effects from adjacent atomic fields( electrons and nuclei ) similar to the subtle shifts that enable nmr to be used to determine part or all of a molecular structure. I am guessing( again! ) that if a single hydrogen atom in isolation could be cycled through an energy transition the emission spectrum would be singular. Take all above with a pinch of salt.
Title: Re: Where does quantization of energy of electromagnetic radiation come from?
Post by: hamdani yusuf on 31/05/2023 09:47:27
Returning to the original question, EMR is not necessarily quantised. Maxwell's propagation laws apply at any and all frequencies, so you can in principle generate radiation at any photon energy you want.

What Planck said was IF you have an ideal particle rattling about in a perfectly elastic box, it can only have discrete energy levels, and you can use this model to predict the UV spectrum etc. If the box does not have defined dimensions then the number of permissible states tends to infinity, hence the black body continuum with a continuous distribution of energy versus wavelength as predicted by the rigid box model for any particular wavelength.
The reason why we ended up with quantum mechanics was because Maxwell's electromagnetic theory doesn't give accurate predictions against experimental results.

Quote
https://en.wikipedia.org/wiki/History_of_quantum_mechanics#Founding_experiments
Founding experiments

Thomas Young's double-slit experiment demonstrating the wave nature of light. (c. 1801)
Henri Becquerel discovers radioactivity. (1896)
J. J. Thomson's cathode ray tube experiments (discovers the electron and its negative charge). (1897)
The study of black-body radiation between 1850 and 1900, which could not be explained without quantum concepts.
The photoelectric effect: Einstein explained this in 1905 (and later received a Nobel prize for it) using the concept of photons, particles of light with quantized energy.
Robert Millikan's oil-drop experiment, which showed that electric charge occurs as quanta (whole units). (1909)
Ernest Rutherford's gold foil experiment disproved the plum pudding model of the atom which suggested that the mass and positive charge of the atom are almost uniformly distributed. This led to the planetary model of the atom (1911).
James Franck and Gustav Hertz's electron collision experiment shows that energy absorption by mercury atoms is quantized. (1914)
Otto Stern and Walther Gerlach conduct the Stern?Gerlach experiment, which demonstrates the quantized nature of particle spin. (1920)
Arthur Compton with Compton scattering experiment (1923)
Clinton Davisson and Lester Germer demonstrate the wave nature of the electron[27] in the electron diffraction experiment. (1927)
Carl David Anderson with the discovery positron (1932), validated Paul Dirac's theoretical prediction of this particle (1928)
Lamb?Retherford experiment discovered Lamb shift (1947), which led to the development of quantum electrodynamics.
Clyde L. Cowan and Frederick Reines confirm the existence of the neutrino in the neutrino experiment. (1955)
Clauss J?nsson's double-slit experiment with electrons. (1961)
The quantum Hall effect, discovered in 1980 by Klaus von Klitzing. The quantized version of the Hall effect has allowed for the definition of a new practical standard for electrical resistance and for an extremely precise independent determination of the fine-structure constant.
The experimental verification of quantum entanglement by John Clauser and Stuart Freedman. (1972)
The Mach?Zehnder interferometer experiment conducted by Paul Kwiat, Harold Wienfurter, Thomas Herzog, Anton Zeilinger, and Mark Kasevich, providing experimental verification of the Elitzur?Vaidman bomb tester, proving interaction-free measurement is possible. (1994)

What does Maxwellian electromagnetism actually predict in each of those experiments? What were the assumptions made in making those predictions? Is it possible to change some of those assumptions which would change the predictions and reconcile with the experiments?
Title: Re: Where does quantization of energy of electromagnetic radiation come from?
Post by: hamdani yusuf on 31/05/2023 10:15:00
In atoms, only certain orbitals are permitted (quantization)
- To calculate these orbitals, you need to solve the wave equation for the electron.
- Some of these orbitals are spherical, but others look like a cluster of balloons assembled by a clown. How do you calculate the radius and centripetal motion for these?

If you want a simple understanding, have a look at Bohr's model of the atom, where an electron's angular momentum is quantized (classical physics has no equivalent).
- Or de Broglie's model where the electron has a wavelength, and that wavelength must have an integer number of wavelengths to be stable (classical physics has no equivalent).
- But for a good model, you have to solve the relativistic Schroedinger equation, which gets quite complex for anything bigger than a hydrogen atom. Even a Hydrogen atom is beyond what they are paying me here!

See: https://en.wikipedia.org/wiki/Atomic_orbital#Bohr_atom

In macroscopic objects, electrically charged particles interact according to Coulomb's law. Is it still working in microscopic objects?
How does it contribute to the formation of atomic orbitals?

The experiment below shows that mass of atomic nucleus affects the emission spectrum, thus it also affects atomic orbitals. Is there any known formula relating them?
http://myslu.stlawu.edu/~jmil/physics/legacy/student_projects/2001/fiacco.shtml
Quote
The Visible Spectrum of Hydrogen vs. Deuterium
Suzanne Fiacco '01

Abstract:

    The purpose of this project is to use a reflection spectrometer to find the differences in wavelengths between the spectrum of hydrogen atoms and the spectrum of deuterium atoms. Hydrogen and deuterium share common characteristics. Deuterium is also known as heavy hydrogen because the weight of deuterium is twice that of hydrogen. Hydrogen is the simplest atom, which consists of one proton and one electron while deuterium is made up of one neutron, one proton, and one electron. Since the physical properties indicate that hydrogen and deuterium are very similar, one would expect their wavelengths to be very similar. In this projects, we calculated three of the visible wavelengths in the hydrogen spectrum to be 656.478 nm, 486.542 nm, and 434.415 nm. For deuterium we calculated that these wavelengths shift to 656.296 nm, 486.409 nm, and 434.295 nm respectively due to the additional mass in the neutron in the nucleus. In doing this experiment we measured the wavelengths in the hydrogen atom to be 656.489 nm, 486.44 nm, and 434.238 nm. For deuterium we measured the visible wavelengths to be 656.295 nm, 486.315 nm, and 434.115 nm. After measuring the intensity verses wavelength for the visible spectrum, we can determine the shift in wavelength for the red, blue, and violet lines as we change the source from hydrogen to deuterium. We conclude that the percent error between the differences in wavelengths between the spectrum of hydrogen and the spectrum of deuterium to be 1.65%, 5.3%, and 2.5% for the wavelengths of red, blue, and violet respectively.

For more information, contact Dr. Catherine Jahncke: cjah@stlawu.edu
Title: Re: Where does quantization of energy of electromagnetic radiation come from?
Post by: paul cotter on 31/05/2023 10:43:26
Maxwell's equations work perfectly well within their limited scope- they are not meant to be a TOE. I think I have already pointed this out.
Title: Re: Where does quantization of energy of electromagnetic radiation come from?
Post by: hamdani yusuf on 31/05/2023 10:54:44
Maxwell's equations work perfectly well within their limited scope- they are not meant to be a TOE. I think I have already pointed this out.
Don't you want to know what makes it not working in microscopic scale?
Or rather, what makes it work in most macroscopic scale?
Note that flat earth model and Geocentric model also work perfectly well within their limited scope- they are not meant to be a TOE.
Title: Re: Where does quantization of energy of electromagnetic radiation come from?
Post by: alancalverd on 31/05/2023 12:44:51
What does Maxwellian electromagnetism actually predict in each of those experiments?
Maxwell's equations describe the propagation of electromagnetic radiation, nothing more or less. 

The lift equation F = 0.5ρAv2 tells you how an aircraft will fly, but nothing about tyre skid on landing. So what?
Title: Re: Where does quantization of energy of electromagnetic radiation come from?
Post by: Bored chemist on 31/05/2023 12:52:47
What does Maxwellian electromagnetism actually predict in each of those experiments?
Maxwell's equations describe the propagation of electromagnetic radiation, nothing more or less. 

The lift equation F = 0.5ρAv2 tells you how an aircraft will fly, but nothing about tyre skid on landing. So what?
Actually, slightly less.
They describe how it travels through a homogeneous medium.
They don't, for example, explain the blue sky.
Title: Re: Where does quantization of energy of electromagnetic radiation come from?
Post by: hamdani yusuf on 31/05/2023 13:26:05
Maxwell's equations describe the propagation of electromagnetic radiation, nothing more or less. 
I'm not limiting the model to Maxwell's equations. It also includes equations for Coulomb force, Lorentz force, Newton's mechanics, and other non-quantum electromagnetic relationships.
Title: Re: Where does quantization of energy of electromagnetic radiation come from?
Post by: hamdani yusuf on 31/05/2023 13:27:01
The lift equation F = 0.5ρAv2 tells you how an aircraft will fly, but nothing about tyre skid on landing. So what?
Maxwell's equations work perfectly well within their limited scope- they are not meant to be a TOE. I think I have already pointed this out.
Don't you want to know what makes it not working in microscopic scale?
Or rather, what makes it work in most macroscopic scale?
Note that flat earth model and Geocentric model also work perfectly well within their limited scope- they are not meant to be a TOE.
Title: Re: Where does quantization of energy of electromagnetic radiation come from?
Post by: alancalverd on 01/06/2023 15:17:37
They don't, for example, explain the blue sky.

Rayleigh scattering in the atmosphere is a classical continuum effect, entirely consistent with Maxwell.
Title: Re: Where does quantization of energy of electromagnetic radiation come from?
Post by: Bored chemist on 01/06/2023 16:36:25
They don't, for example, explain the blue sky.

Rayleigh scattering in the atmosphere is a classical continuum effect, entirely consistent with Maxwell.
I'm fairly sure scattering depends on the size of the scattering centres.
In which case, the medium has to be "lumpy", but Maxwell's eqns don't deal with lumps.
They are consistent, but they won't model it.
Title: Re: Where does quantization of energy of electromagnetic radiation come from?
Post by: alancalverd on 01/06/2023 20:55:04
Nobody said they do. The lift equation doesn't model the ground run correction, but they are entirely consistent with the business of taking off and landing because it's all classical continuum mechanics.
Title: Re: Where does quantization of energy of electromagnetic radiation come from?
Post by: Bored chemist on 01/06/2023 21:25:26
Nobody said they do.

I'm sure someone did...
Maxwell's equations describe the propagation of electromagnetic radiation, nothing more or less. 
Well, it's actually "less". They only describe the propagation of radiation through a vacuum. They don't even explain the blue colour of the sky.
Title: Re: Where does quantization of energy of electromagnetic radiation come from?
Post by: alancalverd on 01/06/2023 23:08:50
Once the scattering has occurred, the spectrally-shifted radiation propagates exactly according to Maxwell because it is electromagnetic radiation.

I seem to have to keep repeating the bloody obvious: Maxwell's equations describe the propagation of EM radiation, not its interactions nor even its generation (the generation of radio waves is easily explained with them but atomic spectra and x-rays need a different approach - quantum mechanics is helpful) .
Title: Re: Where does quantization of energy of electromagnetic radiation come from?
Post by: Bored chemist on 01/06/2023 23:29:25
I seem to have to keep repeating the bloody obvious: Maxwell's equations describe the propagation of EM radiation,
I seem to have to keep pointing out the obvious.
Maxwell's equations only describe the propagation of EM radiation in a vacuum.
And nature abhors a vacuum.
So, exactly where do they apply?
Before you answer that, just pause and consider how big an atom is.
Title: Re: Where does quantization of energy of electromagnetic radiation come from?
Post by: alancalverd on 02/06/2023 08:34:12
Maxwell's equations only describe the propagation of EM radiation in a vacuum.
No. They apply to any medium if you substitute  εm and μm for ε0 and μ0. The vacuum value is admittedly an experimental approximation, but none the worse for that.

To make life easy, we measure and publish dimensionless relative permittivities and permeabilities for various materials (including air and metamaterials) so you can just multiply the vacuum value as appropriate.
Title: Re: Where does quantization of energy of electromagnetic radiation come from?
Post by: Bored chemist on 02/06/2023 08:44:57
It's like swimming in treacle...
Please show me the bit of Maxwell's equations that says "sometimes, some of the light shoots out sideways".
For scattering you need two different values of ε and μ- one of each of the medium that's scattering and one of each for the matrix in which it's suspended.
For example, the air molecules have different values from the "vacuum" which is "between" them.

Please show me which of Maxwells' eqns has two values of μ and ε and a value for the size of the particle.
Title: Re: Where does quantization of energy of electromagnetic radiation come from?
Post by: Eternal Student on 02/06/2023 16:40:59
Hi.

No. They apply to any medium if you substitute  εm and μm for ε0 and μ0.
   I would more or less agree with that.   Let's do the "more" bit first:
You can exhibit the propagation of an e-m wave through a simple material with a speed 1/√(μmεm) < c    when you reformulate Maxwells equation by replacing the E and B fields with D and H fields where necessary.    For a simple dielectric material,  the relationship between the electric polarisation, P, of the material and an underlying E field should be linear (so you will have the constitutive relationship  D  =   εm  E etc.).   Similarly for a simple magnetic material the magnetic susceptibility of the material should also remain constant so that you have a linear constitutive relationship B = μm H between the B and H fields.   
    Overall then,  what you ( @alancalverd ) are saying is that provided you consider "Maxwells equations" to be the version which is often called "Maxwells equations in matter"  rather than "Maxwells equations in a vacuum" then everything works fine and you can exhibit a wave that propagates as required with an appropriate speed for the medium.    I would agree with that - for simple linear materials.

So that brings us to the "less" bit:
To make life easy, we measure and publish dimensionless relative permittivities and permeabilities for various materials (including air and metamaterials) so you can just multiply the vacuum value as appropriate.
    There are some materials for which we just can't - there isn't a simple scalar relating E and D fields  OR  the B and H fields.

    1.  Hard Ferromagnetic material can retain a Magnetisation even when the H field is reduced back to 0 after first being a strong non-zero field in some direction.  So the relationship between  B and H fields inside the material is not the simple linear relationship we would want,  instead it can depend on the history of the fields the material has been exposed to.

    2.  Superconducting materials also have complicated relationships between B and H fields:   For example, in Type-I superconductors we can have Magnetic susceptibility χm = -1 throughout some critical range of the H field (giving a relative permeability μr =  1 + χm = 0 ) but then undergoes a discontinuity and we have  χm = 0   (and hence μr immediately changing to 1)  outside that range.

    3.   Similarly, not all dielectric materials will be simple linear materials (where the D and E fields would be linearly related).   I've not studied it but I have been informed that sometimes the dielectric polarisation of a material isn't even in the same direction as the applied E field (e.g. in some crystalline structures we require a rank-2 tensor, T, to relate the (vector) E field to the (vector) D field:   D  = T E     )

    I would not vouch for how (or even "if") an e-m wave can propagate through some of these non-linear materials.   For all I know, the propagation of an e-m pulse through some non-linear materials could be extremely unusual:
1.  A pulse of e-m radiation sent into the material could travel through the material along path(s) that may not be straight lines.
2.  It may not always take the same path but instead may depend on the prior history of E and B fields that were applied to that material.   Since the e-m wave is itself changing the E and B fields inside the material when it passes through, the first part of the pulse may exit the material in a different place to where later parts of the pulse exit the material.

Best Wishes.
Title: Re: Where does quantization of energy of electromagnetic radiation come from?
Post by: Bored chemist on 02/06/2023 18:09:20
If you get a high enough field strength practically everything behaves non-linearly.
Even air  misbehaves if you try hard enough.
Title: Re: Where does quantization of energy of electromagnetic radiation come from?
Post by: hamdani yusuf on 03/06/2023 06:55:17
If you get a high enough field strength practically everything behaves non-linearly.
Even air  misbehaves if you try hard enough.
I'm wondering how it would look like under different air pressure.
Title: Re: Where does quantization of energy of electromagnetic radiation come from?
Post by: hamdani yusuf on 03/06/2023 06:59:04
Do Maxwell equations explain electrostatic and magnetostatic interactions?
If they don't, what does?
Is it compatible with Maxwell equations?
Title: Re: Where does quantization of energy of electromagnetic radiation come from?
Post by: Bored chemist on 03/06/2023 11:52:59
If you get a high enough field strength practically everything behaves non-linearly.
Even air  misbehaves if you try hard enough.
I'm wondering how it would look like under different air pressure.
Something like this will give you a clue.
http://lamptech.co.uk/Documents/M2%20Pressure.htm
If you get the pressure high enough you will blur out the spectral lines and et what looks like BBR.
Title: Re: Where does quantization of energy of electromagnetic radiation come from?
Post by: alancalverd on 03/06/2023 21:55:08
Do Maxwell equations explain electrostatic and magnetostatic interactions?
If they don't, what does?
Is it compatible with Maxwell equations?
No, they are derived from experiments that show that a moving charge creates a magnetic field and a varying magnetic field induces a voltage in a conductor. These are essentially dynamic phenomena.
Title: Re: Where does quantization of energy of electromagnetic radiation come from?
Post by: alancalverd on 03/06/2023 22:02:16
There are some materials for which we just can't - there isn't a simple scalar relating E and D fields  OR  the B and H fields.
....which is why I said for various materials.

We are quite used to dealing with hysteresis and birefringence. You could still model em propagation in nonlinear materials by saying that εm  is not necessarily constant, but what usually matters in practice is the overall "black box" transfer function of a dense material, not the detail of what happens inside the box.
Title: Re: Where does quantization of energy of electromagnetic radiation come from?
Post by: hamdani yusuf on 04/06/2023 02:22:08
Do Maxwell equations explain electrostatic and magnetostatic interactions?
If they don't, what does?
Is it compatible with Maxwell equations?
No, they are derived from experiments that show that a moving charge creates a magnetic field and a varying magnetic field induces a voltage in a conductor. These are essentially dynamic phenomena.
Afaik, electrostatic force follows Coulomb's law. What about magnetostatic force?
Title: Re: Where does quantization of energy of electromagnetic radiation come from?
Post by: alancalverd on 04/06/2023 19:39:26
Not quite an inverse square law because there is no point source of magnetic field, but Wikipedia sets out all the equations you are likely to need, and when we are dealing with the projectile effect of an MRI magnet on a steel oxygen cylinder, hammer head or nut and bolt, an inverse square law is an adequate approximation from 3 meters distance until the projectile becomes supersonic at about 0.5m from the patient.
Title: Re: Where does quantization of energy of electromagnetic radiation come from?
Post by: paul cotter on 05/06/2023 08:54:27
Hamdani, the biot-savart equation is the counterpart to coulomb, describing the magnetostatic.
Title: Re: Where does quantization of energy of electromagnetic radiation come from?
Post by: hamdani yusuf on 05/06/2023 11:41:12
Hamdani, the biot-savart equation is the counterpart to coulomb, describing the magnetostatic.
Can it describe interaction between two permanent magnets?
How about a magnet and a small ferromagnetic material?
Title: Re: Where does quantization of energy of electromagnetic radiation come from?
Post by: Eternal Student on 05/06/2023 17:01:50
Hi.

Can it describe interaction between two permanent magnets?
How about a magnet and a small ferromagnetic material?

   In general Maxwell's equations are used for Magnetostatics.  As @paul cotter  stated,  some are more relevant than others and the Biot-Savart law is useful.    You can get the Biot-Svart law out of Maxwell's equations with minimal assumptions.

    They won't model the force between permanent magnets and ferromagnetic materials on their own.   You need to make a few additions:
   (i)  Assign a magnetic substance a (vector) Magnetisation  M.   This depends on the properties of the material.   LaTex still isn't working so you can't have any formulas, only words.     It is the vector sum of magnetic dipole moments per unit volume of the material.
    (ii)  You also have to understand what a magnetic dipole is.   In general they can always be modelled as being equivalent to a microscopic loop of current.   (We have terminology like "bound current" to describe the equivalent current that would create the dipole that exists in a place.   Sometimes that can be thought of or identified as an actual current, e.g. an electron whizzing around an atom.  Sometimes it can't.  Bound current is not usually an actual current but just a way of modelling a magnetic dipole).
    (iii)   Make assumptions about how magnets and magnetic materials behave.   The microscopic explanation doesn't necessarily need to concern us (but there is one, with varying degrees of sophistication and accuracy).   For the macroscopic behaviour you only need to know how the Magnetisation,  M,  of a magnetisable material will vary with an applied field.    So, for linear materials we have   M  =  χm H   with  χm = a constant called the magnetic susceptibility.    The H field for a simple linear material is just a linear multiple of the B field,   B = μH  with μ being the permeability of the material (rather than μ0 as for a vacuum).

   I think that's enough additions.   With that you will be able to determine the forces between magnets and/or simple linear magnetic materials.   Basically all magnets are just a collection of magnetic dipoles and magnetic materials become a collection of dipoles in their Prescence.   So the final determination is entirely based on the fields and forces exerted on and by magnetic dipoles.

    I think there was a previous post or two that discussed non-linear materials.  That's more complicated.

   As outlined by @alancalverd ,  dipoles don't follow an inverse square law for the field strength they produce.   At large enough distances, it's an inverse cube law.   (To be honest I'm surprised Alancalverd uses a 1/r2 law as an approximation at any range - but very close it isn't a perfect 1/r3 law,  which we both agree on)
   To further complicate it, in addition to the thing that created the field being a dipole, the thing that is acted on by the field is also a dipole.   So it experiences a force that depends on the gradient of the B field instead of being directly proportional to B (we have F = Bm for a simple monopole of magnetic charge m    but  F = ∇(B.m) for a dipole moment m).   
    So at large distances a dipole experiences a force from another dipole that falls off ~  1/r4.  This assumes that both dipoles were permanent and unchanging dipoles.   If only one had been a permanent magnet and the other is merely a magnetisable material, then  (by the relationship M = χmH )  the magnetisable material becomes less magnetised the further away it gets from the permanent magnet.  So the force existing between the permanent dipole and the magnetisable material falls off with an even higher power (~ 1/r7 ).

I know formulas are not always displaying well.  See  https://en.wikipedia.org/wiki/Force_between_magnets#Magnetic_dipole%E2%80%93dipole_interaction
for some discussion of constructing  the 1/r4  formula for the force between two magnetic dipoles. 

Best Wishes.
Title: Re: Where does quantization of energy of electromagnetic radiation come from?
Post by: paul cotter on 05/06/2023 20:07:19
Thanks for that ES, I was fairly sure Hamdani would be looking for elaboration and you saved me a lot of work and I very much doubt I could have supplied such a clear and concise description. There is one slight addition I would like to make, purely for the general reader. Coulomb and Biot Savart equations look very similar but there is one important difference: the source of these forces is the charge and current respectively- a differential charge can be any charge added or subtracted but a differential current does not exist as current flow requires continuity and only an integral can be used. I haven't expressed that to my satisfaction, didn't sleep much last night and i'm wooly headed.
Title: Re: Where does quantization of energy of electromagnetic radiation come from?
Post by: alancalverd on 05/06/2023 23:09:16
To be honest I'm surprised Alancalverd uses a 1/r2 law as an approximation at any range

It's good enough to persuade clinical folk not to take ferromagnetic objects inside the magnet cage without boring them with divs and curls. I'm sure you could calculate the precise acceleration of a flat-screen television in a 3T field, but my problem was how to extract the debris without scaring the next patient.
Title: Re: Where does quantization of energy of electromagnetic radiation come from?
Post by: hamdani yusuf on 06/06/2023 14:07:24
As outlined by @alancalverd ,  dipoles don't follow an inverse square law for the field strength they produce.   At large enough distances, it's an inverse cube law.   (To be honest I'm surprised Alancalverd uses a 1/r2 law as an approximation at any range - but very close it isn't a perfect 1/r3 law,  which we both agree on)
This article shows the derivation.
Quote
The net force acting between the dipole and point entity X will be:
FD = k X x / (R-δ /2)2  - k X x / (R+δ /2)2
we can rewrite the above in the form:
FD = [kXx/R2] / (1-δ /2R)2 - [kXx/R2] / (1+δ /2R)2
For the condition δ <<2R, which was set as one of our assumptions, we are justified to apply the
binomial approximation (1+x)n
≈ 1+nx, or 1/(1+x)n
≈ 1-nx, valid for x<< 1. This reduces:
1/(1-δ /2R)2 to 1+δ /R, and 1/(1+δ /2R)2 to 1-δ /R
The force field equation can therefore be approximated as:
FD ≈ [kXx/R2](1+δ /R) - [kXx/R2](1-δ /R)
FD ≈ [kXx/R2](1+δ /R - 1 + δ /R)
FD ≈ 2kXxδ /R3 or simply FD ~ 1/R3

https://www.gsjournal.net/h/papers_download.php?id=1833

And the question remains. Classical theories don't tell where the quantization of energy comes from.
Title: Re: Where does quantization of energy of electromagnetic radiation come from?
Post by: Bored chemist on 06/06/2023 14:33:44
I'm sure you could calculate the precise acceleration of a flat-screen television in a 3T field,
Quite possibly zero.
So it experiences a force that depends on the gradient of the B field instead of being directly proportional to B
Title: Re: Where does quantization of energy of electromagnetic radiation come from?
Post by: Bored chemist on 06/06/2023 14:36:27
Classical theories don't tell where the quantization of energy comes from.
In fairness this is true.
It's more or less tautology because, in this context , "classical" means "not quantum".

(Rather than between the Baroque and the Romantic periods)
Title: Re: Where does quantization of energy of electromagnetic radiation come from?
Post by: hamdani yusuf on 07/06/2023 03:34:55
AFAIK, there are at least 2 distinct quantization in electromagnetic radiation.

First is quantization of radiation energy, as expressed by Planck's energy equation E = n.h.f, which was concluded by Planck to justify his formula known as Planck's law which fits the curve from experimental data of black body radiation. In this equation, the frequency can still have any real (positive) value.

The second one is quantization of atomic radiation frequency, which is observed in spectral line emission. Balmer discovered empirical formula to describe the spectral line emissions of the hydrogen atom. Bohr interpreted it as the evidence for the existence of atomic orbitals.
Title: Re: Where does quantization of energy of electromagnetic radiation come from?
Post by: hamdani yusuf on 07/06/2023 08:32:29
Quote
https://en.wikipedia.org/wiki/Planck_postulate

The Planck postulate (or Planck's postulate), one of the fundamental principles of quantum mechanics, is the postulate that the energy of oscillators in a black body is quantized, and is given by

E = n h ν
where

n is an integer (1, 2, 3, ...),
h is Planck's constant, and
ν  (the Greek letter nu, not the Latin letter v) is the frequency of the oscillator.

The postulate was introduced by Max Planck in his derivation of his law of black body radiation in 1900. This assumption allowed Planck to derive a formula for the entire spectrum of the radiation emitted by a black body. Planck was unable to justify this assumption based on classical physics; he considered quantization as being purely a mathematical trick, rather than (as is now known) a fundamental change in the understanding of the world.[1] In other words, Planck then contemplated virtual oscillators.

In 1905, Albert Einstein adapted the Planck postulate to explain the photoelectric effect, but Einstein proposed that the energy of photons themselves was quantized (with photon energy given by the Planck?Einstein relation), and that quantization was not merely a feature of microscopic oscillators. Planck's postulate was further applied to understanding the Compton effect, and was applied by Niels Bohr to explain the emission spectrum of the hydrogen atom and derive the correct value of the Rydberg constant.

For convenience in typing, I'll just use f for frequency.
Planck's energy equation E = n.h.f
This energy equation is equivalent with power equation, if we take another parameter j as h/t.
P = E/t = n.h/t.f = n.j.f
while h has a unit of Joule.second/cycle, the unit of j is Joule/cycle.
Increasing radiation power can be done be adding more oscillators, which can only be done discretely.

I think this is more intuitive, for following reason. Suppose we have a radiation source so dim that n=1 and f=1 Hz. Minimum value for E=h Joule. But radiation power is still undetermined. If it's radiated in 1 second then the power is h Watt. If it's radiated in 1000 second, then the power is h milliWatt.
The unit of Joule/cycle for j is more intuitive than Joule.second/cycle for h.
If a radiation source oscillates for 1 cycle, then the radiated energy is j Joules. If it oscillates for 1000 cycles, then the radiated energy is 1000.j Joules.
Title: Re: Where does quantization of energy of electromagnetic radiation come from?
Post by: Bored chemist on 07/06/2023 08:36:51
I think this is more intuitive,
Nobody else seems to.
Title: Re: Where does quantization of energy of electromagnetic radiation come from?
Post by: hamdani yusuf on 07/06/2023 14:45:49
I think this is more intuitive,
Nobody else seems to.
Does it mean I'm wrong? We haven't heard someone else's opinion about this.

I think this is more intuitive, for following reason. Suppose we have a radiation source so dim that n=1 and f=1 Hz. Minimum value for E=h Joule. But radiation power is still undetermined. If it's radiated in 1 second then the power is h Watt. If it's radiated in 1000 second, then the power is h milliWatt.
In currently more common used form of equation, radiation power is not quantized, due to unrestricted time period. Note that quantization of power equation is mathematically equivalent, but it has no flexibility to change the time period.
This type of quantization reminds me of Millikan's oil drop experiment to determine the electric charge of single electron. They can only change in a system by a discrete amount.
Title: Re: Where does quantization of energy of electromagnetic radiation come from?
Post by: Bored chemist on 07/06/2023 15:15:28
In currently more common used form of equation, radiation power is not quantized,
And that's fair enough because power isn't quantised in the way that energy is.
Power isn't even a conserved quantity.
Title: Re: Where does quantization of energy of electromagnetic radiation come from?
Post by: hamdani yusuf on 08/06/2023 03:50:26
Power isn't even a conserved quantity.
When power changes, where does the difference go? Does it just appear/disappear? Or Is it merely converted into something else?
Planck got the equation from his research on black body radiation, which can be reasonably generalized to electromagnetic radiation. There's no adequate justification to extrapolate it to other type of power or energy, such as gravitational potential energy.
Title: Re: Where does quantization of energy of electromagnetic radiation come from?
Post by: Bored chemist on 08/06/2023 08:57:02
When power changes, where does the difference go?
You tell me.
I charge a battery slowly at a low power overnight, then, in the morning, I use that battery to deliver a lot of power briefly to start my car.

Where did the "extra" power come from?

Essentially any time we talk about "energy storage" we are using it to change power.
Title: Re: Where does quantization of energy of electromagnetic radiation come from?
Post by: Bored chemist on 08/06/2023 09:02:15
There's no adequate justification to extrapolate it to other type of power or energy, such as gravitational potential energy.
Do you know that gravity affects photon energy?
Title: Re: Where does quantization of energy of electromagnetic radiation come from?
Post by: hamdani yusuf on 08/06/2023 12:05:31
There's no adequate justification to extrapolate it to other type of power or energy, such as gravitational potential energy.
Do you know that gravity affects photon energy?
Afaik, it changes the energy by affecting the frequency, which doesn't affect the Planck energy equation, nor it's quantization.
Title: Re: Where does quantization of energy of electromagnetic radiation come from?
Post by: Bored chemist on 08/06/2023 13:39:15
Afaik, it changes the energy by affecting the frequency
The statement is meaningless.
How could it affect one but not the other?
The statement is meaningless.
Title: Re: Where does quantization of energy of electromagnetic radiation come from?
Post by: alancalverd on 08/06/2023 23:58:32
Planck's energy equation E = n.h.f,
Where does the n come from? E = hf in my universe. But that doesn't mean energy is quantised. Planck's model describes the energy levels available to a particle constrained in a box. Obviously if you have two particles with the same frequency in the box, you have twice as much energy, but the box can be any size you like and a free particle can have any amount of energy you care to give it.

So haviong establioshed, for the umpteenth time, that energy is not necessarily quantised, we have reduced the question "where does the quantisation come from" tothe same level of pointlessness as "why are unicorns born feet-first?"
Title: Re: Where does quantization of energy of electromagnetic radiation come from?
Post by: hamdani yusuf on 09/06/2023 03:27:47
Where does the n come from? E = hf in my universe.
Are you living in a different universe?
Some sources mention that n is the number of photons.
A universe containing only 1 photon doesn't allow anyone to exist, let alone to think.
https://www.ux1.eiu.edu/~cfadd/1160/Ch28QM/Planck.html
Quote
Planck's Hypothesis
In 1900 Max Planck proposed a formula for the intensity curve which did fit the experimental data quite well. He then set out to find a set of assumptions -- a model -- that would produce his formula. Instead of allowing energy to be continuously distributed among all frequencies, Planck's model required that the energy in the atomic vibrations of frequency f was some integer times a small, minimum, discrete energy,

Emin = hf
where h is a constant, now known as Planck's constant,

h = 6.626176 x 10-34 J s
Planck's proposal, then requires that all the energy in the atomic vibrations with frequency f can be written as

E = n h f
where n in an integer, n = 1, 2, 3, . . . No other values of the energy were allowed. The atomic oscillators could not have energy of (2.73) hf or (5/8) hf.

This idea that something -- the energy in this case -- can have only certain discrete values is called quantization. We say that the energy is quantized. This is referred to as Planck's quantum hypothesis. "Quantum" means how great or of a fixed size.

This article mentions near its end.
Quote
1 photon -> 1 electron
1 electron -> 1 photon
http://www.phys.ttu.edu/~slee/3301/2018_Fall/F18_3301_Lecture4.pdf
Title: Re: Where does quantization of energy of electromagnetic radiation come from?
Post by: hamdani yusuf on 09/06/2023 03:40:07
Afaik, it changes the energy by affecting the frequency
The statement is meaningless.
How could it affect one but not the other?
The statement is meaningless.
In energy equation E=n.h.f, you can change E with constant f by changing n.
Not all form of energy involve frequency. Gravitational potential energy and electrostatic potential energy has no frequency in their equations.
Title: Re: Where does quantization of energy of electromagnetic radiation come from?
Post by: hamdani yusuf on 09/06/2023 08:23:21
Where does the n come from? E = hf in my universe. But that doesn't mean energy is quantised. Planck's model describes the energy levels available to a particle constrained in a box. Obviously if you have two particles with the same frequency in the box, you have twice as much energy, but the box can be any size you like and a free particle can have any amount of energy you care to give it.

So haviong establioshed, for the umpteenth time, that energy is not necessarily quantised, we have reduced the question "where does the quantisation come from" tothe same level of pointlessness as "why are unicorns born feet-first?"

Planck hypothesized about quantization of radiation energy, that's what's being discussed here. Other types of energy can be mentioned for comparison.
You don't seem to accept currently mainstream interpretation of quantum theory. Perhaps you can share the sources that you think more representative to your thinking.
Title: Re: Where does quantization of energy of electromagnetic radiation come from?
Post by: Bored chemist on 09/06/2023 08:33:52
Some sources mention that n is the number of photons.
Which sources?
A universe containing only 1 photon doesn't allow anyone to exist, let alone to think.
Nobody said it did.
Title: Re: Where does quantization of energy of electromagnetic radiation come from?
Post by: Bored chemist on 09/06/2023 08:35:57
Gravitational potential energy and electrostatic potential energy has no frequency in their equations.
Nor do they refer to a single entity. You need  a "system" of at least two particles for gravitational or electrical potential to exist.

That's a separate class of problems.

Title: Re: Where does quantization of energy of electromagnetic radiation come from?
Post by: Bored chemist on 09/06/2023 08:37:05
In energy equation E=n.h.f, you can change E with constant f by changing n.
Only if there's an interaction with something else.
Title: Re: Where does quantization of energy of electromagnetic radiation come from?
Post by: alancalverd on 09/06/2023 08:53:36
hf is the energy of a photon. Obviously if you have more photons in a box, you have more energy in the box. That doesn't mean that energy is quantised, only that photons are.
Title: Re: Where does quantization of energy of electromagnetic radiation come from?
Post by: hamdani yusuf on 09/06/2023 10:20:20
hf is the energy of a photon. Obviously if you have more photons in a box, you have more energy in the box. That doesn't mean that energy is quantised, only that photons are.
Can you add radiation energy by half a photon?
Title: Re: Where does quantization of energy of electromagnetic radiation come from?
Post by: Bored chemist on 09/06/2023 12:27:22
hf is the energy of a photon. Obviously if you have more photons in a box, you have more energy in the box. That doesn't mean that energy is quantised, only that photons are.
Can you add radiation energy by half a photon?
No.
Because, as Alan said, phonons are quantised.
Title: Re: Where does quantization of energy of electromagnetic radiation come from?
Post by: alancalverd on 09/06/2023 14:16:34
You could ad a photon with 0.5f to the box, but it would still be a whole photon.
Title: Re: Where does quantization of energy of electromagnetic radiation come from?
Post by: hamdani yusuf on 09/06/2023 14:54:40
You could ad a photon with 0.5f to the box, but it would still be a whole photon.
It means the radiation energy in particular frequency is quantized. It can be interpreted as photon, but it's not the only possible interpretation. Planck himself didn't interpret it as such.
Title: Re: Where does quantization of energy of electromagnetic radiation come from?
Post by: alancalverd on 09/06/2023 22:55:53
No. It means that a quantum of radiation has a particular energy, and that energy is directly related to the frequency of  the wave  that models the propagation of that quantum.

My car is blue. That doesn't mean that all blue cars are mine, or that all blue things are cars.
Title: Re: Where does quantization of energy of electromagnetic radiation come from?
Post by: hamdani yusuf on 10/06/2023 07:02:39
No. It means that a quantum of radiation has a particular energy, and that energy is directly related to the frequency of  the wave  that models the propagation of that quantum.
The concept of quantum of radiation was created after the discovery of quantization in radiation energy.
That quantization was thought to be necessary to explain the energy distribution of black body radiation.
You are getting it reversed.
Title: Re: Where does quantization of energy of electromagnetic radiation come from?
Post by: alancalverd on 10/06/2023 11:37:51
The entire history of science is essentially the history of the gradual retreat of human stupidity in the face of improving models of the universe, but it has no bearing on how the universe behaves.

The fact is that the energy of electromagnetic radiation in general can take any value (up to the mass energy equivalent of the entire universe) but the energy of a single photon (the means by which we model the interaction of em energy with matter)  is hf where f is the frequency of the wave that models that photon's propagation.
Title: Re: Where does quantization of energy of electromagnetic radiation come from?
Post by: Eternal Student on 11/06/2023 04:27:23
Hi.

That quantization was thought to be necessary to explain the energy distribution of black body radiation.
   You (Hamdani) have said something like this in a few places.

If we back up to post #83 for a moment,   I think the section you had in quotations presented Planck's early assumptions and his model for Blackbody radiation much more accurately.

    Planck didn't really suggest all e-m radiation was quantised and it certainly wasn't required to explain Blackbody radiation.    Taking an extract from your quotation - he only required that the energy in the atomic vibrations of frequency f was some integer times a small, minimum, discrete energy,.   To paraphrase that -  all Planck advocated for was quantised oscillators in the walls.

   I've edited and stripped out most of what I was originally going to say.   This is a condensed version:
Blackbody radiation is usually modelled by considering radiation that can exist inside a cavity.  So Planck would only require the radiation that can be supported inside a cavity to be quantised  (have allowed energies of the form nhω  for n an integer).   There are many ways and many assumptions you can make to obtain or justify that result.   For example, you can do exactly what Planck was thought to have done originally:  Assume the radiation was generated by some sort of microscopic "oscillators" within the walls of a cavity.   If those oscillators have energy which is quantised that will be sufficient to ensure the radiation inside the cavity at every frequency will have energy that is also quantised.
   So Planck wanted quantised oscillators in his cavity walls only.   Radiation in general or that exists elsewhere in the world (not inside a cavity at thermal equilibrium with cavity walls) was under no obligation to be quantised.

   That may seem like a lot of words to point out a very small difference.   It wasn't done just to be annoying but instead just to clarify what Planck needed to assume and what was not needed.   Yes, you can derive the blackbody spectrum by assuming radiation is quantised but you just don't have to and historically it does not look like Planck made that assumption.   All he needed was quantised oscillators in the walls, the quantisation of any radiation that could be found inside the cavity would follow automatically.   This has some relevance for suggesting one reason why all bits of light you might find anywhere (e.g. not just inside a cavity) would still be quantised:   If every bit of light (anywhere) has it origin ultimately in a transition that happened in an atom, then the source of all e-m radiation is suitably quantised,  so all the radiation you will find will automatically be suitably quantised.

     To check or examine this, you'd want to try to find light that you know was not made by any transition (or quantised change) within an atom (including the nucleus for very high frequency e-m radiation) and just check if this bit of light is still quantised in the same way that we expect.   The only bit of light I can think of whose origin cannot be ultimately traced back to an atom would be light from the big bang, or from any time before we had nucleogenesis and atoms in existence.   However, until the era of recombination when atoms were formed and continuing to form, the universe remained opaque and light just did not travel through it.    So there just isn't any of that light that you can see, if indeed there was ANY light (travelling e-m waves of some frequency) as we understand it that early in the universe.   Some sources of information will just directly state that the era of recombination was the first moment of light existing:   This was the moment of first light in the universe, between 240,000 and 300,000 years after the Big Bang, known as the Era of Recombination.     [ https://phys.org/news/2016-11-universe.html ].   The existence of e-m radiation and of atoms that produce or absorb only discrete quanta of e-m radiation has been inextricably linked since the earliest of days.

Best Wishes.
Title: Re: Where does quantization of energy of electromagnetic radiation come from?
Post by: hamdani yusuf on 11/06/2023 10:38:51
Planck didn't really suggest all e-m radiation was quantised and it certainly wasn't required to explain Blackbody radiation.    Taking an extract from your quotation - he only required that the energy in the atomic vibrations of frequency f was some integer times a small, minimum, discrete energy,.   To paraphrase that -  all Planck advocated for was quantised oscillators in the walls.

Quote
In 1900, the British physicist Lord Rayleigh derived the λ−4 dependence of the Rayleigh?Jeans law based on classical physical arguments, relying upon the equipartition theorem. This law predicted an energy output that diverges towards infinity as wavelength approaches zero (as frequency tends to infinity). Measurements of the spectral emission of actual black bodies revealed that the emission agreed with Rayleigh's calculation at low frequencies but diverged at high frequencies; reaching a maximum and then falling with frequency, so the total energy emitted is finite. Rayleigh recognized the unphysical behavior of his formula at high frequencies and introduced an ad hoc cutoff to correct it, but experimentalists found that his cutoff did not agree with data.[1][3] Hendrik Lorentz also presented a derivation of the wavelength dependence in 1903. More complete derivations, which included the proportionality constant, were presented in 1905 by Rayleigh and Sir James Jeans and independently by Albert Einstein.[3] Rayleigh believed that this discrepancy could be resolved by the equipartition theorem failing to be valid for high-frequency vibrations, while Jeans argued that the underlying cause was matter and luminiferous aether not being in thermal equilibrium.[3]
https://en.m.wikipedia.org/wiki/Rayleigh%E2%80%93Jeans_law
Planck's quantization was an ad hoc correction to Rayleigh's model which fortunately fit with experimental data. His argument was that it takes more energy to produce higher frequency radiation, which provides the cut off he needs to explain black body radiation curve.
Title: Re: Where does quantization of energy of electromagnetic radiation come from?
Post by: Bored chemist on 11/06/2023 10:47:20
The concept of quantum of radiation was created after the discovery of quantization in radiation energy.
That's linguistically unlikely.
So Planck wanted quantised oscillators in his cavity walls only.   Radiation in general or that exists elsewhere in the world (not inside a cavity at thermal equilibrium with cavity walls) was under no obligation to be quantised.
That's impossible.
How could quantised radiation escape the box other than 1 quantum at a time?
The box isn't allowed to lose half a quantum, because that would leave and impossible "half quantum" behind.
Title: Re: Where does quantization of energy of electromagnetic radiation come from?
Post by: hamdani yusuf on 11/06/2023 11:27:45
All he needed was quantised oscillators in the walls, the quantisation of any radiation that could be found inside the cavity would follow automatically.   This has some relevance for suggesting one reason why all bits of light you might find anywhere (e.g. not just inside a cavity) would still be quantised:   If every bit of light (anywhere) has it origin ultimately in a transition that happened in an atom, then the source of all e-m radiation is suitably quantised,  so all the radiation you will find will automatically be suitably quantised.
Some e-m radiations come from molecular activities. Some others come from macroscopic mechanical vibration.
Title: Re: Where does quantization of energy of electromagnetic radiation come from?
Post by: Bored chemist on 11/06/2023 12:08:07
Some e-m radiations come from molecular activities. Some others come from macroscopic mechanical vibration.
Has em radiation from a  macroscopic vibration ever been detected?
Title: Re: Where does quantization of energy of electromagnetic radiation come from?
Post by: Eternal Student on 11/06/2023 13:34:59
Hi.

    The phrase "don't shoot the messenger" comes to mind.   I am not the one saying that all Planck originally assumed was quantised oscillators in the walls.   The history books are saying that. 
     However, later versions of the derivation for the blackbody spectrum will often assume that it was the radiation inside the cavity itself that was directly quantised.  So you can find many derivations in many textbooks that do directly make that assumption and they may even be better or superior derivations of the result.   We're all in agreement that we now have enough evidence to suggest that radiation is quantised (everywhere).   For example, Einstein's work on the photo-electric effect did suggest that.   However, what we now assume and may use in a modern derivation for the Blackbody spectrum does NOT change what Planck originally assumed and how he started to explain the spectrum.

       Planck assumed that the sources of radiation are atoms in a state of oscillation and that the vibrational energy of each oscillator may have any of a series of discrete values but never any value between.
[Taken from Encyclopedia Britannica:  https://www.britannica.com/science/Plancks-radiation-law]

      Planck, in his treatment to solve blackbody radiation, considered that the electromagnetic waves inside the cavity of the blackbody are standing waves due to oscillating charges on the wall of the cavity. At equilibrium, the frequency of oscillation of the oscillating charge is equal to the frequency of the electromagnetic wave produced by it i.e. at equilibrium, energy of the oscillating charge is equal to the energy of the produced electromagnetic wave.
   
https://physics.stackexchange.com/questions/703294/plancks-assumptions-for-treatment-of-blackbody-radiation

     Max Planck concentrated on modelling the oscillating charges that must exist in the oven walls, radiating heat inwards and, in thermodynamic equilibrium, themselves being driven by the radiation field. He found he could account for the observed curve if he required these oscillators not to radiate energy continuously, as the classical theory would demand, but they could only lose or gain energy in chunks, called quanta, of size hν, for an oscillator of frequency ν
https://chem.libretexts.org/Courses/Pacific_Union_College/Quantum_Chemistry/01%3A_The_Dawn_of_the_Quantum_Theory/1.02%3A_Quantum_Hypothesis_Used_for_Blackbody_Radiation_Law

    Then there's the source of information that Hamdani presented himself in post #83.  Just read that carefully and you will see it is talking about the oscillators in the walls.

This is a non-exhaustive list of information sources.  Do your own historical research and you will turn up more evidence.    It really does seem that Planck's original assumptions focused on the oscillators in the walls of the cavity.   The quantisation of the radiation that was inside the cavity just followed as a consequence of that.   Indeed, the very first derivation proposed by Planck didn't offer much explanation for the quantisation what-so-ever,  there may not have been any explanation on a microscopic scale.   He just noted that if you assume the energy was quantised then you do get a spectrum which matches with the experimental results well.   I'm not a expert on the History of science but it was shortly after that when his first attempts at providing a microscopic explanation was made - those early attempts focused on quantising the oscillators in the walls.

As stated in my earlier post: 
Yes, you can derive the blackbody spectrum by assuming radiation is quantised but you just don't have to and historically it does not look like Planck made that assumption.   All he needed was quantised oscillators in the walls, the quantisation of any radiation that could be found inside the cavity would follow automatically.

- - - - - - - - - - - - - -

Some e-m radiations come from molecular activities. Some others come from macroscopic mechanical vibration.
   As @Bored chemist  started to suggest - that is not actually obvious.   While we can model radio waves as being something that might be produced in an aerial just by charges being accelerated up and down the aerial,  it is still almost certainly just a model.   Each individual photon (or bit of radio wave) probably is being produced by some interaction(s) within atoms (or molecules with hundreds of interacting atoms) i.e. as a consequence of some fundamental atomic scale production mechanism.

Best Wishes.
Title: Re: Where does quantization of energy of electromagnetic radiation come from?
Post by: hamdani yusuf on 12/06/2023 10:01:46
Some e-m radiations come from molecular activities. Some others come from macroscopic mechanical vibration.
Has em radiation from a  macroscopic vibration ever been detected?
Something like this.
Title: Re: Where does quantization of energy of electromagnetic radiation come from?
Post by: hamdani yusuf on 12/06/2023 10:05:09
However, what we now assume and may use in a modern derivation for the Blackbody spectrum does NOT change what Planck originally assumed and how he started to explain the spectrum.
I wonder how can anyone disagree with this.
Title: Re: Where does quantization of energy of electromagnetic radiation come from?
Post by: Bored chemist on 12/06/2023 12:50:24
Something like this.
I see no photons.
Title: Re: Where does quantization of energy of electromagnetic radiation come from?
Post by: Bored chemist on 12/06/2023 12:54:57
I suspect that the reason this is such a  long thread is that the subject is this "Where does quantization of energy of electromagnetic radiation come from?".
And essentially that's asking why quantisation happens.
In general, science isn't good at saying "why" things happen.
We can put forward models of what we expect to happen; but that's not teh same thing.
I'm not sure that anyone can really explain why an apple falls from the tree.
We can say that that everything attracts everything, but we can't say why everything attracts everything.

You can make up an explanation in terms of gravitons and that's great- it even sort-of explains an inverse square law.

But it still doesn't say why gravitons exist.


Title: Re: Where does quantization of energy of electromagnetic radiation come from?
Post by: alancalverd on 12/06/2023 14:47:45
I think the question is back to front. It is sensible to ask why a quantum change of energy levels produces electromagnetic radiation,and the answer is quite simply "conservation", but not every em spectrum is necessarily quantised.
Title: Re: Where does quantization of energy of electromagnetic radiation come from?
Post by: Eternal Student on 12/06/2023 17:26:03
Hi.

I wonder how can anyone disagree with this.
   i.d.k.      Do some historical research,  find some document showing Planck actually proceeded in some other way initially.

      I'm not an expert in the History of science.    I've just spent a bit of time looking at Blackbody radiation because some things have worried me, specifically I couldn't see why some simplifications work as well as they did etc.   So I made some efforts to find several variations in the derivation and to try and separate what Planck did initially from the more modern re-workings of the derivations that can be found in textbooks.
   This thread:  https://www.thenakedscientists.com/forum/index.php?topic=86164.0   appeared some time ago - although the topic changed into a discussion of something else at the end.   You (Hamdani) were one of the frequent contributors to the post towards the end.   There wasn't a definitive conclusion reached and at least one aspect of the typical derivation of the Blackbody spectrum still seems unsatisfactory to me,  it seems like a bit of a fudge or cheat.   I can't get any LaTeX support on this forum so there's no easy way to communicate that bit of the derivation and it's also not especially relevant to this forum thread.

Best Wishes.
Title: Re: Where does quantization of energy of electromagnetic radiation come from?
Post by: Bored chemist on 12/06/2023 17:29:29
but not every em spectrum is necessarily quantised.
OK, once you have quantised energy states then conservation explains why they gain or lose energy in quanta.

Why is any of them quantised?
Why does a hydrogen atom have quantised energy?
Title: Re: Where does quantization of energy of electromagnetic radiation come from?
Post by: alancalverd on 12/06/2023 22:54:27
Why does a hydrogen atom have quantised energy?
It's all down to Heisenberg and Schrodinger. For other atoms, you also need a teaspoon of Pauli.
Title: Re: Where does quantization of energy of electromagnetic radiation come from?
Post by: Bored chemist on 12/06/2023 22:57:59
Why does a hydrogen atom have quantised energy?
It's all down to Heisenberg and Schrodinger.
No.
Grass was green before either of them was born.
Title: Re: Where does quantization of energy of electromagnetic radiation come from?
Post by: hamdani yusuf on 12/06/2023 23:39:06
I found some old videos explaining how Planck came up with his law.


Title: Re: Where does quantization of energy of electromagnetic radiation come from?
Post by: alancalverd on 13/06/2023 09:19:43
Why does a hydrogen atom have quantised energy?
It's all down to Heisenberg and Schrodinger.
No.
Grass was green before either of them was born.
I smell a philosopher! Indeed, only God can answer the question "why", and as there is no God, the question is meaningless.  But if you want to know "how", I refer to the three hon gents I mentioned earlier.

As for the color of grass, I was blissfully unaware that hydrogen had such a strong absorption in the visible red spectrum that it makes grass look green. I thought it was something to do with the molecular orbitals of chlorophyll (Schrodinger and Pauli again - and the clue is in the name!)  Clearly my botanist brother in law has been misleading me.
Title: Re: Where does quantization of energy of electromagnetic radiation come from?
Post by: Bored chemist on 13/06/2023 13:24:04
Ho hum...
I thought it was something to do with the molecular orbitals
Yes.
And we model the molecular orbitals as combinations of atomic orbitals.
And we model atomic orbitals as if they are "hydrogen like atoms".
So, if hydrogen energy levels weren't quantised, the orbitals of the other atoms wouldn't be.
And if the other atoms' orbitals were not quantised, then the orbitals of molecules wouldn't be.
And if the energy levels of molecules weren't quantised  then grass wouldn't be green.

But we know that grass was green before Heisenberg and Schrodinger were born.
So we know that quantisation is not a consequence of anything they did, said or were.

So we know that you were wrong.

Why not just accept it next time and save bandwidth?


Incidentally, hot hydrogen atoms do have a strong absorption for red light.
It's the hydrogen alpha line looked at from the other point of view.
Title: Re: Where does quantization of energy of electromagnetic radiation come from?
Post by: alancalverd on 13/06/2023 13:31:16
So we all look forward to your explanation without using anything that Heisenberg, Schrodinger or Pauli wrote.

And we model the molecular orbitals as combinations of atomic orbitals.
Which would produce some very odd-shaped molecules indeed. Whatever happened to delocalisation since 1963?
Title: Re: Where does quantization of energy of electromagnetic radiation come from?
Post by: Bored chemist on 13/06/2023 15:04:09
So we all look forward to your explanation without using anything that Heisenberg, Schrodinger or Pauli wrote
I already gave it.
Science can't tell you "why" something happens.

I'm seeking to explain why the thread is going nowhere and I didn't need to mention any physicists to do so.


Whatever happened to delocalisation since 1963?
I presume that among the things that happened was that you forgot how it works.
When people talk about things like pi bonds and sigma bonds they are talking about the (dominant) atomic orbitals from which they are formed.
Things like "sp2 hybrid" even tell you what proportions you need to mix.
Title: Re: Where does quantization of energy of electromagnetic radiation come from?
Post by: Eternal Student on 13/06/2023 17:33:05
Hi.

About the videos:
   I've only glanced at some bits of the videos you (Hamdani) presented.   They seem to support the idea that Planck focused on the oscillators in the walls.  See, for example,  around 6:10 ~ 6:30 in the first video where the narrator states that light came from those quantised oscillators. 

   I found at least one statement that was made which isn't necessarily true.   In the second video at about 7:00 to 7:30  the narrator discusses the use of a cuboidal shaped cavity for the mathematical model (which is the typical choice).   There is a fairly casual statement made that any shaped cavity could have been used, the result would be the same but the maths would just be harder.   That is not at all obvious and I don't think it should be stated as established fact.  There are several adjustments to the method that would normally be made.  In particular, there are some extra assumptions that seem essential and must be added to the mathematical model to correctly predict that very small and peculiarly shaped cavities will still ultimately produce the usual BB spectrum.
   There are some pieces of equipment which work purely because there is a cavity (including a suitable material for the walls of the cavity) with the right shape and dimensions to seriously influence the spectrum or frequency of radiation that is supported.   (See, for example, the discussion of lasers, resonance and cavities  in this post  https://www.thenakedscientists.com/forum/index.php?topic=86164.msg700593#msg700593   which you (Hamdani) seemed to approve of at the time).   
 
   The videos do seem to take a fairly conventional approach and walk you through how you must have stationary e-m waves that fit inside the cavity,   only these will be considered as supported "modes" of radiation.    Overall, the mathematical model discussed (which is a typical derivation for the BB spectrum) is just an approximation but the narrator doesn't make it apparent where all of those approximations are made.   For example, at time 8:05 to 8:30 in the second video a formula for the density of modes is said to be "not easy to find" and just written down.   This is important because it's only this approximation which allows us to finish with a continuous spectrum rather than just a discrete set of frequencies.    So, it's important to note that the entire method, even for simple cuboidal shaped cavities, is just an approximation.   As suggested earlier, you need some additional approximations or assumptions before you could be certain that an arbitrary shaped cavity would still give rise to the usual BB spectrum.

[LATE EDITING:  This post is already long, I've removed further discussion about the mathematical model(s) used for deriving the BB spectrum in very small and peculiarly shaped cavities.   There's no need to side-track the thread too much].

Best Wishes.
Title: Re: Where does quantization of energy of electromagnetic radiation come from?
Post by: Bored chemist on 13/06/2023 19:01:21
 I found at least one statement that was made which isn't necessarily true.   In the second video at about 7:00 to 7:30  the narrator discusses the use of a cuboidal shaped cavity for the mathematical model (which is the typical choice).   There is a fairly casual statement made that any shaped cavity could have been used, the result would be the same but the maths would just be harder.   That is not at all obvious and I don't think it should be stated as established fact.
Imagine that you wanted to model a BB of some other shape.
You could make it out of small cube shaped BB (for which the maths is known) an then recognise that, if they are all at the same temperature, there's no net radiation exchange between the cubes; only at the surface.
And you can then rescale the problem - effectively using a different unit of length- for example, you could use cubic centimetre cubes to fill up a six foot sphere and calculate the wavelengths in fathoms.
Title: Re: Where does quantization of energy of electromagnetic radiation come from?
Post by: hamdani yusuf on 13/06/2023 23:43:52
Incidentally, hot hydrogen atoms do have a strong absorption for red light.
It's the hydrogen alpha line looked at from the other point of view.
How hot or cold does it take to produce absorption spectrum?
(https://www.thenakedscientists.com/forum/proxy.php?request=http%3A%2F%2Fwww.4college.co.uk%2Fas%2Fel%2Fhow.gif&hash=5a8aff28e0cfb2dad7c616eb9d3374b6)
http://www.4college.co.uk/as/el/how.gif
(https://www.daviddarling.info/images/types_of_spectra.jpg)
https://www.daviddarling.info/images/types_of_spectra.jpg

(https://sites.ualberta.ca/~pogosyan/teaching/ASTRO_122/lect6/figure05-14.jpg)
https://sites.ualberta.ca/~pogosyan/teaching/ASTRO_122/lect6/figure05-14.jpg
Title: Re: Where does quantization of energy of electromagnetic radiation come from?
Post by: hamdani yusuf on 14/06/2023 06:18:37
In particular, there are some extra assumptions that seem essential and must be added to the mathematical model to correctly predict that very small and peculiarly shaped cavities will still ultimately produce the usual BB spectrum.
Can you specify what are they?
Title: Re: Where does quantization of energy of electromagnetic radiation come from?
Post by: hamdani yusuf on 14/06/2023 08:06:59
I smell a philosopher! Indeed, only God can answer the question "why", and as there is no God, the question is meaningless.  But if you want to know "how", I refer to the three hon gents I mentioned earlier.
Learning is basically information collection, filtration, and compression process.
When we find a physical observation, we can accept that as the natural process of involved objects in that particular situation. Alternatively, we can try to find underlying basic principles which can be used to reproduce that observation, given some reasonable model and assumptions.
Keppler's laws of planetary motions essentially compressed Tycho's table of planetary observations. Newton's universal gravitation compressed them even further.
Title: Re: Where does quantization of energy of electromagnetic radiation come from?
Post by: Bored chemist on 14/06/2023 08:32:28
Incidentally, hot hydrogen atoms do have a strong absorption for red light.
It's the hydrogen alpha line looked at from the other point of view.
How hot or cold does it take to produce absorption spectrum?

How strongly do you want it to absorb?

Hot enough to significantly populate the first excited state.
How come you don't know that?

Title: Re: Where does quantization of energy of electromagnetic radiation come from?
Post by: hamdani yusuf on 14/06/2023 12:55:49
How strongly do you want it to absorb?

Hot enough to significantly populate the first excited state.
How come you don't know that?
Strong enough to be detected by common phone camera unambiguously.
Can it be done under room temperature?

Your statement suggests that there's a minimum temperature limit to show the absorption spectrum, contrary to the pictures which suggest that there's a maximum temperature limit instead. What's your reference?
Title: Re: Where does quantization of energy of electromagnetic radiation come from?
Post by: hamdani yusuf on 14/06/2023 13:25:06
Incidentally, hot hydrogen atoms do have a strong absorption for red light.
It's the hydrogen alpha line looked at from the other point of view.
How hot or cold does it take to produce absorption spectrum?
(https://www.thenakedscientists.com/forum/proxy.php?request=http%3A%2F%2Fwww.4college.co.uk%2Fas%2Fel%2Fhow.gif&hash=5a8aff28e0cfb2dad7c616eb9d3374b6)
http://www.4college.co.uk/as/el/how.gif
(https://www.daviddarling.info/images/types_of_spectra.jpg)
https://www.daviddarling.info/images/types_of_spectra.jpg

(https://sites.ualberta.ca/~pogosyan/teaching/ASTRO_122/lect6/figure05-14.jpg)
https://sites.ualberta.ca/~pogosyan/teaching/ASTRO_122/lect6/figure05-14.jpg
Instead of absorption, the pictures look more like scattering effect.
I've tried to find the video version of the experiment shown in the pictures above. Unfortunately I can't find it on YouTube. I can only find some experiment of hydrogen emission spectrum using hydrogen lamp.
But I'll take it as an opportunity to make the video myself. Let me hear your suggestions and advices.

To produce hydrogen gas, I think I can use electrolysis of dilute sodium hydroxide solution. Note that pure water doesn't conduct electricity well, while salt water produces dangerous chlorine gas as byproduct.

Torricelli's method using tall tube can be used to create partial vacuum or low pressure gas.

Sunlight focussed by a convex lens or concave mirror can be used as the bright light source.
Title: Re: Where does quantization of energy of electromagnetic radiation come from?
Post by: Bored chemist on 14/06/2023 14:59:11
Sodium carbonate is less of a hazard than the hydroxide but in either case you will be here until doomsday trying to scrub salt spray out of the hydrogen or you will get a yellow flame.

How strongly do you want it to absorb?

Hot enough to significantly populate the first excited state.
How come you don't know that?
Strong enough to be detected by common phone camera unambiguously.
Can it be done under room temperature?

Your statement suggests that there's a minimum temperature limit to show the absorption spectrum, contrary to the pictures which suggest that there's a maximum temperature limit instead. What's your reference?
You have not understood the pictures.
You seem to have read "cooler gas" as if it says "cool gas". Quite often, the "cool" gas is an oxyacetylene flame.
The only requirement there is that it's cooler than the hot black-body which is providing the radiation.

Your statement suggests that there's a minimum temperature limit to show the absorption spectrum
As I already pointed out.
It depends.
It depends on the answer to this
How strongly do you want it to absorb?
And you failed to answer it. Why is that?

This tells you about flame absorption spectra.
https://www.agilent.com/en/support/atomic-spectroscopy/atomic-absorption/flame-atomic-absorption-instruments/how-does-aas-work-aas-faqs
Title: Re: Where does quantization of energy of electromagnetic radiation come from?
Post by: Eternal Student on 14/06/2023 18:32:10
Hi.

Thanks for your time and suggestion @Bored chemist .     I've spent a bit of time considering your idea.    I've also written two replies but none of them were short.   This is the third attempt with brevity as high priority.

You could make it out of small cube shaped BB (for which the maths is known)
    The general idea of building a new (bigger) shape by joining some shapes which you already know will produce a BB spectrum together (face to face etc.) is reasonably sound.

And you can then rescale the problem - effectively using a different unit of length-
    Yes, there are some places in the usual mathematical method for deriving the BB spectrum formula where scale is fairly arbitrary.   The main problem is just that you could be magnifying the errors in the approximations that are made in that method.   
   
     The standard mathematical method for deriving the Planck formula pivots around assuming two things that are actually contradictory (an assumption for the first half, a different one for the second half):   1.  Start by assuming you will only have only a discrete set of supported modes.  You need this to estimate a "Density of modes" function.
    2.  Now you assume you have a continuous distribution of supported frequencies which is given by this "density of modes" function.   Stop assuming you had a discrete distribution, throw that away.

    Those two steps or assumptions are only compatible or consistent enough  when  L (the size of the cavity) is large or λ (the wavelength of a supported mode) is small.   While we can argue that scale is arbitrary at some places in the mathematics, we do want to stay in some sensible region (of physical length L and wavelengths λ) where we can believe that both halves of the method will be reasonably true simultaneously.   (i.e. both assumptions are reasonable representations of what Nature is actually doing).
- - - - - - - - - - -
Can you specify what they are?    (extra assumptions for small cavities etc.)
    One of the things you'd like to do is to reduce the discrete nature of the frequencies for the supported modes and make them more spread out or continuous immediately.
    The spectrum we observe from experiment (even for small cavities) does tend to be fairly smooth and continuous rather than having well defined spikes like a discrete spectrum.  So it is clear that Nature is not doing exactly what the mathematical model assumed (about fitting standing waves in the cavity etc. - as in step1 or the first half of the method).    One adjustment we can make to the mathematical model is to relax the boundary conditions for the standing waves slightly   (the boundary conditions are that the E and B fields are 0 at the walls,  which was represented in your videos by insisting the wave returns to 0 displacement at the walls).   This modification is reasonably justified on physical grounds....
    Specifically, it seems that Nature does NOT see the wall of a cavity as a sharply defined hard wall.   For example, not all of the radiation is stopped or bounced back at the inner edge of the wall,  radiation of some frequencies may penetrate the wall slightly before being absorbed or bounced back etc.   Indeed that does happen and is verifiable in other experiments - e.g. X rays can penetrate most materials to some depth.
    We can fairly easily imagine that if we allow some penetration, then if a wave of precisely λ wavelength would have fitted inside the cavity, then we can actually assume some continuous range of wavelengths around λ are supported.
    We can do even more than this to further justify some more supported wavelengths:  We don't need to consider photons in the model but it's just easier to have this in your mind for a moment and imagine an e-m wave as some collection of photons where each photon could get bounced back at different places into the wall.   It provides some physical grounds to justify having a wave that is incident on the wall be reflected in small pieces and interesting ways at more than just one depth into the wall.  With the principle of superposition we can then maintain standing waves inside the cavity with a wavelength that is much longer than (twice) the length of the cavity.   This helps to mitigate the cut-off or maximum wavelength for supported modes normally governed by the length of the cavity (see above, λmax = 2L ).
       This post is already too long so I've got to stop.   Basically, for very small cavities, we need to adjust assumptions in the mathematical model.   One of the most sensible adjustments is to allow more complicated support of modes (standing waves) inside the cavity (for example allowing penetration into the wall).

Best Wishes.
Title: Re: Where does quantization of energy of electromagnetic radiation come from?
Post by: hamdani yusuf on 15/06/2023 01:56:03
And you failed to answer it. Why is that?
It looks like you fail to find my answer.
Strong enough to be detected by common phone camera unambiguously.
Title: Re: Where does quantization of energy of electromagnetic radiation come from?
Post by: hamdani yusuf on 15/06/2023 02:28:24
This tells you about flame absorption spectra.
https://www.agilent.com/en/support/atomic-spectroscopy/atomic-absorption/flame-atomic-absorption-instruments/how-does-aas-work-aas-faqs
The sample shown in the article is Pb. Turning it into gas requires high temperature. If the element is already gaseous in room temperature, like hydrogen, the flame doesn't seem to be necessary.
The flame is not shown in the pictures I posted either.
Title: Re: Where does quantization of energy of electromagnetic radiation come from?
Post by: hamdani yusuf on 15/06/2023 02:55:05
Specifically, it seems that Nature does NOT see the wall of a cavity as a sharply defined hard wall.   For example, not all of the radiation is stopped or bounced back at the inner edge of the wall,  radiation of some frequencies may penetrate the wall slightly before being absorbed or bounced back etc.   Indeed that does happen and is verifiable in other experiments - e.g. X rays can penetrate most materials to some depth.
Also, if the wall is not a good electric conductor, radio wave can penetrate it. I discussed the limitations of photon model in explaining electromagnetic radiation in another thread.
Title: Re: Where does quantization of energy of electromagnetic radiation come from?
Post by: Bored chemist on 15/06/2023 08:40:17
And you failed to answer it. Why is that?
It looks like you fail to find my answer.
Strong enough to be detected by common phone camera unambiguously.

Opops!
Sorry, I missed that.
But I'm not sure it would be practically possible to detect with a phone camera, even under optimal conditions.
Title: Re: Where does quantization of energy of electromagnetic radiation come from?
Post by: Bored chemist on 15/06/2023 08:45:56
This tells you about flame absorption spectra.
https://www.agilent.com/en/support/atomic-spectroscopy/atomic-absorption/flame-atomic-absorption-instruments/how-does-aas-work-aas-faqs
The sample shown in the article is Pb. Turning it into gas requires high temperature. If the element is already gaseous in room temperature, like hydrogen, the flame doesn't seem to be necessary.
The flame is not shown in the pictures I posted either.
Heating has three effects.
One is converting the material to vapour- which is not important for hydrogen.
Another is converting the material into atoms (rather than molecules).
That's important for hydrogen, but not with (for example) mercury.

And the third thing heating it does is raise some of the atoms to their first excited state.
The ground state of hydrogen atoms doesn't absorb visible light.
Title: Re: Where does quantization of energy of electromagnetic radiation come from?
Post by: hamdani yusuf on 16/06/2023 05:54:13
Instead of absorption, the pictures look more like scattering effect.
It's unfortunate that the diagram doesn't mention temperature requirements for the depicted phenomena to be observed. Although the spectrum looks like Balmer series, which implies that the gas is hydrogen.
(https://sites.ualberta.ca/~pogosyan/teaching/ASTRO_122/lect6/figure05-14.jpg)
Title: Re: Where does quantization of energy of electromagnetic radiation come from?
Post by: hamdani yusuf on 18/06/2023 01:55:28
Something like this.
I see no photons.
The video shows em radiation caused by macroscopic motion.
I didn't mention any photon. I didn't expect anyone to see it either.
Title: Re: Where does quantization of energy of electromagnetic radiation come from?
Post by: hamdani yusuf on 18/06/2023 02:01:38
Instead of absorption, the pictures look more like scattering effect.
It's unfortunate that the diagram doesn't mention temperature requirements for the depicted phenomena to be observed. Although the spectrum looks like Balmer series, which implies that the gas is hydrogen.
(https://sites.ualberta.ca/~pogosyan/teaching/ASTRO_122/lect6/figure05-14.jpg)
Before I saw this diagram, I wondered where does the absorbed energy go. I guessed that some are transformed into heat, which is then dissipated to the environment.
Title: Re: Where does quantization of energy of electromagnetic radiation come from?
Post by: Bored chemist on 18/06/2023 10:15:20
Instead of absorption, the pictures look more like scattering effect.
There's some similarity but...
There are two big differences.
Absorption warms up the absorber.
The timescales are different.
Scattering is much faster.


A less obvious difference is the spectrum. Shorter wavelengths are much more strongly scattered and all wavelengths are scattered at least slightly.
Title: Re: Where does quantization of energy of electromagnetic radiation come from?
Post by: Bored chemist on 18/06/2023 10:16:06
Before I saw this diagram, I wondered where does the absorbed energy go. I guessed that some are transformed into heat, which is then dissipated to the environment.
Some is.
Title: Re: Where does quantization of energy of electromagnetic radiation come from?
Post by: hamdani yusuf on 18/06/2023 17:14:28
The second one is quantization of atomic radiation frequency, which is observed in spectral line emission. Balmer discovered empirical formula to describe the spectral line emissions of the hydrogen atom. Bohr interpreted it as the evidence for the existence of atomic orbitals.

Quote
In atomic physics, the Bohr model or Rutherford?Bohr model of the atom, presented by Niels Bohr and Ernest Rutherford in 1913, consists of a small, dense nucleus surrounded by orbiting electrons. It is analogous to the structure of the Solar System, but with attraction provided by electrostatic force rather than gravity. In the history of atomic physics, it followed, and ultimately replaced, several earlier models, including Joseph Larmor's solar system model (1897), Jean Perrin's model (1901),[2] the cubical model (1902), Hantaro Nagaoka's Saturnian model (1904), the plum pudding model (1904), Arthur Haas's quantum model (1910), the Rutherford model (1911), and John William Nicholson's nuclear quantum model (1912). The improvement over the 1911 Rutherford model mainly concerned the new quantum mechanical interpretation introduced by Haas and Nicholson, but forsaking any attempt to explain radiation according to classical physics.
https://en.m.wikipedia.org/wiki/Bohr_model
Bohr's model was proposed because then classical physicists thought that accelerating electrons must radiate em wave. But experiments with superconductor shows that electric current can flow in circular motion without radiating away its energy. It's generally accepted that superconductivity is created by forming Cooper pairs.
IMO, it can happen because the radiation from one electron is cancelled out by its pair.
Title: Re: Where does quantization of energy of electromagnetic radiation come from?
Post by: Bored chemist on 18/06/2023 17:19:27
Bohr's model was proposed because then classical physicists thought that accelerating electrons must radiate em wave.
Bohr's model was proposed in spite of classical physics saying that accelerating electrons must radiate em wave.
Title: Re: Where does quantization of energy of electromagnetic radiation come from?
Post by: Bored chemist on 18/06/2023 17:22:55
But experiments with superconductor shows that electric current can flow in circular motion without radiating away its energy.
You don't need superconductors to run into that problem.

An ordinary  magnet should be impossible for much the same reason.
Title: Re: Where does quantization of energy of electromagnetic radiation come from?
Post by: hamdani yusuf on 19/06/2023 07:55:33
You don't need superconductors to run into that problem.

An ordinary  magnet should be impossible for much the same reason.
In superconductor, we can observe the electric current making a macroscopic trajectory.
Title: Re: Where does quantization of energy of electromagnetic radiation come from?
Post by: hamdani yusuf on 28/09/2023 11:10:28

This sun sparkling water looks like a quantization of sunlight, although each point should be much brighter than a single photon. I first realized this phenomenon during a flight above a sea, when the airplane is about to land.

Here's another one.
https://youtube.com/shorts/QCE9-DU0xDg?feature=shared

PS. this forum doesn't support youtube shorts preview, so you'll have to open the link in a new tab/window.
Title: Re: Where does quantization of energy of electromagnetic radiation come from?
Post by: Origin on 28/09/2023 12:40:06
This sun sparkling water looks like a quantization of sunlight,
But of course it isn't really quantized.
Title: Re: Where does quantization of energy of electromagnetic radiation come from?
Post by: alancalverd on 28/09/2023 13:25:02
This sun sparkling water looks like a quantization of sunlight, although each point should be much brighter than a single photon.
You can model the effect as multiple samples of a continuum. No evidence of or requirement for quantisation in the macroscopic observation. As the water waves are moving smoothly, any apparent reflection can also appear to move smoothly - there are no discrete vectors or forbidden transitions.   
Title: Re: Where does quantization of energy of electromagnetic radiation come from?
Post by: hamdani yusuf on 28/09/2023 14:27:23
What makes the sun reflection in the video below less sparkling?
Title: Re: Where does quantization of energy of electromagnetic radiation come from?
Post by: alancalverd on 28/09/2023 17:05:04
Different sun position.
Title: Re: Where does quantization of energy of electromagnetic radiation come from?
Post by: hamdani yusuf on 29/09/2023 02:58:13
Different sun position.
Do you mean reflection angle?

Let's compare it with some othr videos.




IMO, the sparkling reflections occur when the sky is clear, which makes stark contrast between the sun and the background. When the sky is cloudy, the contrast in brightness is reduced.
Title: Re: Where does quantization of energy of electromagnetic radiation come from?
Post by: alancalverd on 29/09/2023 12:21:44
"Point sparkles" obviously require a point source (or a rare event where the wavelets are all substantially concave) and the source must be behind the observer.

Diffuse source, or source in front of the observer, produces diffuse reflections in most cases.
Title: Re: Where does quantization of energy of electromagnetic radiation come from?
Post by: hamdani yusuf on 29/09/2023 13:58:05
"Point sparkles" obviously require a point source (or a rare event where the wavelets are all substantially concave) and the source must be behind the observer.

Diffuse source, or source in front of the observer, produces diffuse reflections in most cases.
Seen from the surface of the earth, the sun is not like a point source. It has a significant angular diameter. But somehow the sparkles seem like being quantized.
Title: Re: Where does quantization of energy of electromagnetic radiation come from?
Post by: alancalverd on 29/09/2023 19:21:49
Compared with the angle subtended at the observer's eye by the ocean, it is a point
Title: Re: Where does quantization of energy of electromagnetic radiation come from?
Post by: hamdani yusuf on 30/09/2023 04:07:09
Compared with the angle subtended at the observer's eye by the ocean, it is a point
What do you mean? Is the angle close to zero?

(https://upload.wikimedia.org/wikipedia/commons/thumb/6/64/Subtended_angle.svg/330px-Subtended_angle.svg.png)
Quote
https://en.wikipedia.org/wiki/Subtended_angle
In geometry, an angle is subtended by an arc, line segment or any other section of a curve when its two rays pass through the endpoints of that arc, line segment or curve section. Conversely, the arc, line segment or curve section confined within the rays of an angle is regarded as the corresponding subtension of that angle. It is also sometimes said that an arc is intercepted or enclosed by that angle.

The precise meaning varies with context. For example, one may speak of the angle subtended by an arc of a circle when the angle's vertex is the centre of the circle.
Title: Re: Where does quantization of energy of electromagnetic radiation come from?
Post by: Origin on 30/09/2023 14:26:59
But somehow the sparkles seem like being quantized.
This is clearly not an example of quantization, perhaps you do not know what is meant by the term quantization?
Title: Re: Where does quantization of energy of electromagnetic radiation come from?
Post by: alancalverd on 30/09/2023 23:53:34
What do you mean? Is the angle close to zero?
The diameter of the sun is about 700,000 km, at a distance of about 140,000,000 km so it subtends an angle of about 0.3 degrees at the observer's eye. At 2 m height, the ocean horizon is about 5,000 m so it subtends an angle of about 89 degrees. That is the difference between "almost a point" and "almost a plane".
Title: Re: Where does quantization of energy of electromagnetic radiation come from?
Post by: hamdani yusuf on 03/10/2023 03:03:44
What do you mean? Is the angle close to zero?
The diameter of the sun is about 700,000 km, at a distance of about 140,000,000 km so it subtends an angle of about 0.3 degrees at the observer's eye. At 2 m height, the ocean horizon is about 5,000 m so it subtends an angle of about 89 degrees. That is the difference between "almost a point" and "almost a plane".
Let's compare that to the moon.



IMO, your explanation is still incomplete.
Title: Re: Where does quantization of energy of electromagnetic radiation come from?
Post by: alancalverd on 03/10/2023 08:54:19
You also need to consider the texture of the reflecting surface.
Title: Re: Where does quantization of energy of electromagnetic radiation come from?
Post by: hamdani yusuf on 03/10/2023 11:11:05
You also need to consider the texture of the reflecting surface.
Right. To produce sparkling, the water surface must not be flat. It can be modeled as combination of many convex and concave mirrors with various curvature. Those convex mirrors produce smaller images of the reflected object compared to flat mirrors. For observer further away than the focal point, concave mirrors also produce smaller images.
(https://www.deepsentinel.com/wp-content/uploads/2023/02/Convex-vs.-Concave.png)
But these are not enough to explain the sparkling effect.
My hypothesis is that the sparkling effect is caused by the granular nature of the sensors, including digital cameras and human eyes. It can be tested by comparing the sun reflection on waving surface of water using cameras with different resolution, and camera obscura with different sizes.
Title: Re: Where does quantization of energy of electromagnetic radiation come from?
Post by: alancalverd on 03/10/2023 13:09:58
The third image in #153 shows both "sparkles" and "general glow", so nothing to do with the resolution of the imaging system.
Title: Re: Where does quantization of energy of electromagnetic radiation come from?
Post by: Origin on 03/10/2023 17:30:47
But these are not enough to explain the sparkling effect.
I think it perfectly explains the effect. 
Do you now realize that the sparkling is not quantized?
Title: Re: Where does quantization of energy of electromagnetic radiation come from?
Post by: hamdani yusuf on 04/10/2023 08:31:22
But these are not enough to explain the sparkling effect.
I think it perfectly explains the effect. 
Do you now realize that the sparkling is not quantized?
We know for sure that electronic sensors are spatially quantized, although it isn't necessarily true that every pixel has the same size or sensitivity.
Do you think it has no observable effect?
Title: Re: Where does quantization of energy of electromagnetic radiation come from?
Post by: alancalverd on 04/10/2023 10:14:25
Not at the level shown in your images. It was sometimes difficult to distinguish individual leaves on a whole tree photographed with a 56k camera but the blobs in your images are a lot bigger, and in any case the question is about quantisation of the radiation, not the image receptor!
Title: Re: Where does quantization of energy of electromagnetic radiation come from?
Post by: hamdani yusuf on 04/10/2023 11:12:40
The third image in #153 shows both "sparkles" and "general glow", so nothing to do with the resolution of the imaging system.
It's visibly zoomed in. Which makes an area of the moon surface will fall into larger area on the sensor. This reduces the sparkling effect.
The curvature of water surface is quite randomly distributed from concave, flat and convex plane. When it's a little bit concave, the focal point can be quite far away from the surface, and even get closer to the sensor. It can produce an effect like zooming in.
Title: Re: Where does quantization of energy of electromagnetic radiation come from?
Post by: hamdani yusuf on 04/10/2023 11:15:51
Not at the level shown in your images. It was sometimes difficult to distinguish individual leaves on a whole tree photographed with a 56k camera but the blobs in your images are a lot bigger, and in any case the question is about quantisation of the radiation, not the image receptor!
The radiation power follows inverse square law of distance. And the effect from quantization of distance is insignificant in this case.
Title: Re: Where does quantization of energy of electromagnetic radiation come from?
Post by: Origin on 04/10/2023 13:25:49
We know for sure that electronic sensors are spatially quantized, although it isn't necessarily true that every pixel has the same size or sensitivity.
Do you think it has no observable effect?
I think that the sparkling of water is not quantized.  I also think that once you get an idea in your head no amount of evidence or logic can convince you otherwise.
Title: Re: Where does quantization of energy of electromagnetic radiation come from?
Post by: hamdani yusuf on 04/10/2023 13:38:54
We know for sure that electronic sensors are spatially quantized, although it isn't necessarily true that every pixel has the same size or sensitivity.
Do you think it has no observable effect?
I think that the sparkling of water is not quantized.  I also think that once you get an idea in your head no amount of evidence or logic can convince you otherwise.
The same applies to you too. You're already convinced that texture of reflecting surface alone is enough to produce sparkling effects as shown in the videos, which I've also observed myself with my own eyes.
Title: Re: Where does quantization of energy of electromagnetic radiation come from?
Post by: Origin on 04/10/2023 15:36:10
The same applies to you too.
And everyone else.  You seem to be the only one that thinks there is some sort of magical thing going on when it is pretty obvious we are just looking at reflections off of the water.

So are you going to acknowledge that the 'sparkling water' is not an example of quantization?  I think there is a possibility that if we pursue this for a couple of posts it could be resolved. 
Title: Re: Where does quantization of energy of electromagnetic radiation come from?
Post by: hamdani yusuf on 05/10/2023 07:51:08
This one shows Sun Reflection on Pacific Ocean from Space.
Title: Re: Where does quantization of energy of electromagnetic radiation come from?
Post by: hamdani yusuf on 05/10/2023 07:56:00
This one is in swimming pool.
Title: Re: Where does quantization of energy of electromagnetic radiation come from?
Post by: alancalverd on 05/10/2023 09:04:23
Thus demonstrating the overriding and practically incalculable importance of water in determining the temperature of the surface of the planet, but nothing else.
Title: Re: Where does quantization of energy of electromagnetic radiation come from?
Post by: hamdani yusuf on 05/10/2023 09:56:57
The same applies to you too.
And everyone else.  You seem to be the only one that thinks there is some sort of magical thing going on when it is pretty obvious we are just looking at reflections off of the water.

So are you going to acknowledge that the 'sparkling water' is not an example of quantization?  I think there is a possibility that if we pursue this for a couple of posts it could be resolved. 
My hypothesis can be falsified easily by showing the same reflection of the sun by rippling water surface using cameras with different resolution, or pin hole cameras with different sizes.
What would it take to falsify your conviction?
Title: Re: Where does quantization of energy of electromagnetic radiation come from?
Post by: alancalverd on 05/10/2023 12:07:25
Measurement of the energy of a solar photon (about 3eV) and the energy radiated from any  individual reflection that you have photographed.

Title: Re: Where does quantization of energy of electromagnetic radiation come from?
Post by: Bored chemist on 05/10/2023 13:44:02
This sun sparkling water looks like a quantization of sunlight,
In what way?

Are you saying the intensity is quantised or the wavelength or what?
Title: Re: Where does quantization of energy of electromagnetic radiation come from?
Post by: hamdani yusuf on 06/10/2023 04:41:32
This sun sparkling water looks like a quantization of sunlight,
In what way?

Are you saying the intensity is quantised or the wavelength or what?

The sparkles look bright and then suddenly disappear instead of fade away.
Title: Re: Where does quantization of energy of electromagnetic radiation come from?
Post by: Bored chemist on 06/10/2023 11:33:51
This sun sparkling water looks like a quantization of sunlight,
In what way?

Are you saying the intensity is quantised or the wavelength or what?

The sparkles look bright and then suddenly disappear instead of fade away.
I guess you can call a video frame rate some sort of "quantisation", but it's a weird use of the idea.
Title: Re: Where does quantization of energy of electromagnetic radiation come from?
Post by: alancalverd on 06/10/2023 15:31:20
Quantisation, sampling, digitisation and scintillation are not the same thing.

Light consists of quanta. Reflected light may scintillate. Any camera samples light. Some digitise the intensity of the sample.
Title: Re: Where does quantization of energy of electromagnetic radiation come from?
Post by: hamdani yusuf on 06/10/2023 23:21:11
I guess you can call a video frame rate some sort of "quantisation", but it's a weird use of the idea.
Do you have a better terminology?

Quantisation, sampling, digitisation and scintillation are not the same thing.
Title: Re: Where does quantization of energy of electromagnetic radiation come from?
Post by: Origin on 07/10/2023 01:58:34
This is so tedious, I'm out.
Title: Re: Where does quantization of energy of electromagnetic radiation come from?
Post by: alancalverd on 07/10/2023 10:41:26
Quote from: Bored chemist on Yesterday at 11:33:51
I guess you can call a video frame rate some sort of "quantisation", but it's a weird use of the idea.
Do you have a better terminology?
Read the second sentence of reply 173, which is explicit. 
Title: Re: Where does quantization of energy of electromagnetic radiation come from?
Post by: hamdani yusuf on 07/10/2023 11:23:06
Let's make a list of things that affect the sparkle.
Brightness of the light source.
Contrast with the background.
Visual size of the light source.
Size of the ripple on reflecting surface.

Angle of reflection doesn't seem to affect much, since the sparkling effect can be seen in both low and high angles.

The effects from pixel size of the sensor is yet to be determined.
Title: Re: Where does quantization of energy of electromagnetic radiation come from?
Post by: hamdani yusuf on 07/10/2023 11:40:28
Read the second sentence of reply 173, which is explicit. 
Do you mean it's the same phenomenon as scintillation?
Title: Re: Where does quantization of energy of electromagnetic radiation come from?
Post by: hamdani yusuf on 07/10/2023 11:49:12
The effects from pixel size of the sensor is yet to be determined.
I just recorded a video of reflected sunlight on a rippling water surface. It looks sparkling when seen using naked eyes. But my phone camera doesn't show that sparkling, either from further or closer distance. Approximately 3 meters vs 1 meters from water surface.  My main camera has 50 MP resolution.
Title: Re: Where does quantization of energy of electromagnetic radiation come from?
Post by: hamdani yusuf on 07/10/2023 13:33:35
Let's make a list of things that affect the sparkle.
Brightness of the light source.
Contrast with the background.
Visual size of the light source.
Size of the ripple on reflecting surface.

Angle of reflection doesn't seem to affect much, since the sparkling effect can be seen in both low and high angles.

The effects from pixel size of the sensor is yet to be determined.
I also think that the sparkling effect involves saturation of sensor pixels. Which is why dimmer light sources don't produce sparkling.
Title: Re: Where does quantization of energy of electromagnetic radiation come from?
Post by: hamdani yusuf on 07/10/2023 13:35:36
This is so tedious, I'm out.
You are free to jump in or out of the discussion here, anytime you like. You don't need anyone's permission.
Title: Re: Where does quantization of energy of electromagnetic radiation come from?
Post by: Bored chemist on 07/10/2023 13:58:42
I guess you can call a video frame rate some sort of "quantisation", but it's a weird use of the idea.
Do you have a better terminology?
Yes.
Frame rate.
Title: Re: Where does quantization of energy of electromagnetic radiation come from?
Post by: alancalverd on 07/10/2023 17:07:13
Reply 180 at least moves towards a definition of sparkling, though whether the visual effect is due to saturation of the sensor, scatter in the sensor, or flare in the lens assembly, is not clear.
Title: Re: Where does quantization of energy of electromagnetic radiation come from?
Post by: hamdani yusuf on 07/10/2023 22:50:16
I guess you can call a video frame rate some sort of "quantisation", but it's a weird use of the idea.
Do you have a better terminology?
Yes.
Frame rate.
It seems like you have never starred at sparkling water surface.
Title: Re: Where does quantization of energy of electromagnetic radiation come from?
Post by: hamdani yusuf on 07/10/2023 22:51:55
Reply 180 at least moves towards a definition of sparkling, though whether the visual effect is due to saturation of the sensor, scatter in the sensor, or flare in the lens assembly, is not clear.
Some sparkling bright spots don't show any flare.
Title: Re: Where does quantization of energy of electromagnetic radiation come from?
Post by: alancalverd on 08/10/2023 14:23:30
In that case you are seeing a scintillating reflection with no receiver distortion.
Title: Re: Where does quantization of energy of electromagnetic radiation come from?
Post by: Bored chemist on 08/10/2023 14:54:04
It seems like you have never starred at sparkling water surface.
It may seem that way to you. But there's no evidence for your view.
Title: Re: Where does quantization of energy of electromagnetic radiation come from?
Post by: hamdani yusuf on 08/10/2023 22:04:46
I guess you can call a video frame rate some sort of "quantisation", but it's a weird use of the idea.
Do you have a better terminology?
Yes.
Frame rate.
https://youtube.com/shorts/QCE9-DU0xDg?feature=shared
Some sparkling dots in the short video last much longer than a frame duration.
Title: Re: Where does quantization of energy of electromagnetic radiation come from?
Post by: Bored chemist on 09/10/2023 12:03:25
Do you understand detector saturation?
Title: Re: Where does quantization of energy of electromagnetic radiation come from?
Post by: alancalverd on 09/10/2023 13:32:32
Some sparkling dots in the short video last much longer than a frame duration.
Which is hardly surprising since each originates from a relatively slowly-changing source.

Semiconductor  detectors should unsaturate in much less than a frame interval, and movie film frames are completely independent, but analog TV tubes and organic eyeballs do exhibit persistence.

You may get the occasional "dead pixel" or synchronous downstream interference that generates a persistent single white dot in a digital camera, but it won't move across the image field and you'd need a large cluster before it becomes conspicuous.
Title: Re: Where does quantization of energy of electromagnetic radiation come from?
Post by: hamdani yusuf on 10/10/2023 02:56:14
Do you understand detector saturation?
Yes. That's what you should say, instead of scan rate.
Title: Re: Where does quantization of energy of electromagnetic radiation come from?
Post by: Bored chemist on 10/10/2023 08:02:31
Do you understand detector saturation?
Yes. That's what you should say, instead of scan rate.
I didn't say scan rate.

Perhaps you should "cut to the chase" and tell us how sparkles on the water would look without "quantisation".
Title: Re: Where does quantization of energy of electromagnetic radiation come from?
Post by: hamdani yusuf on 11/10/2023 05:19:10
Do you understand detector saturation?
Yes. That's what you should say, instead of scan rate.
I didn't say scan rate.

Perhaps you should "cut to the chase" and tell us how sparkles on the water would look without "quantisation".

Ok, you said frame rate.
I apologize.

You can compare the videos I posted. Some of them don't show sparkling effect. I've also recorded my own videos, which will be uploaded when I have time to spare.
Title: Re: Where does quantization of energy of electromagnetic radiation come from?
Post by: hamdani yusuf on 11/10/2023 05:59:48
Meanwhile, Nobel Prize in physics has been announced.



Is the attosecond laser quantized?
How is the quantization characterized?
What's the frequency and intensity profile?
Title: Re: Where does quantization of energy of electromagnetic radiation come from?
Post by: Bored chemist on 11/10/2023 07:56:36
Is the attosecond laser quantized?
Yes.
One laser makes sense as a concept
And 12 lasers makes sense too.
But 1.7 lasers or pi lasers is not meaningful.
Title: Re: Where does quantization of energy of electromagnetic radiation come from?
Post by: hamdani yusuf on 12/10/2023 04:28:08
The thumbnail in third video depicts the laser pulse to have 6.5 number of waves.
How many waves was actually created in the experiment?
What's the real frequency?
What's the real intensity?

If the laser source is moving closer to the observer at high speed, would the pulse width become even shorter?
Title: Re: Where does quantization of energy of electromagnetic radiation come from?
Post by: hamdani yusuf on 19/10/2023 15:42:00
Here's another video trying to explain the experiment.

How Physicists Took An Electron's Picture - Physics Nobel Prize 2023 Explained

Quote
The 2023 Nobel Prize for Physics was awarded to a fantastic trio working towards imaging electrons on the attosecond scale. I am an optical physicist, so I naturally want to take a deeper look at why this discovery is important, how it actually works, and what it unlocks for our understanding of the universe around us and how it actually behaves.

0:00 Electrons and the world of the minute.
1:22 "Everything in physics starts with Einstein" - Isaac Newton
2:26 Breaking the 6 femtosecond record
5:27 How to build the world's fastest laser pulses
7:05 Ad read
7:50 How to see an Electron
9:03 Why don't you just use a single photon?
The video shows that there are multiple photons used, although it's not clear how many.
Title: Re: Where does quantization of energy of electromagnetic radiation come from?
Post by: Bored chemist on 19/10/2023 17:14:39
What's the real frequency?
Badly defined because of the uncertainty principle.
Title: Re: Where does quantization of energy of electromagnetic radiation come from?
Post by: hamdani yusuf on 21/10/2023 09:08:02
BTW, here's my first video showing the sparkling water.
Title: Re: Where does quantization of energy of electromagnetic radiation come from?
Post by: hamdani yusuf on 21/10/2023 09:16:17
What's the real frequency?
Badly defined because of the uncertainty principle.
Can a single photon have more than one frequency?
Title: Re: Where does quantization of energy of electromagnetic radiation come from?
Post by: alancalverd on 21/10/2023 18:23:44
No. E = hν
Title: Re: Where does quantization of energy of electromagnetic radiation come from?
Post by: Bored chemist on 21/10/2023 19:16:03
What's the real frequency?
Badly defined because of the uncertainty principle.
Can a single photon have more than one frequency?
Yes, because of the uncertainty principle.
Title: Re: Where does quantization of energy of electromagnetic radiation come from?
Post by: hamdani yusuf on 22/10/2023 11:09:41
No. E = hν
Yes, because of the uncertainty principle.
I don't think that both answers can be correct.
Title: Re: Where does quantization of energy of electromagnetic radiation come from?
Post by: hamdani yusuf on 22/10/2023 11:20:08
Here's a recent video explaining about uncertainty principle, and it's relationship with Fourier transform.
Title: Re: Where does quantization of energy of electromagnetic radiation come from?
Post by: alancalverd on 22/10/2023 22:50:33
Thanks to indeterminacy you can't predict the energy of a given photon, nor the time at which it will arrive at a point,  with absolute accuracy, but as far as the photon is concerned it only has one energy.
Title: Re: Where does quantization of energy of electromagnetic radiation come from?
Post by: Bored chemist on 23/10/2023 13:28:30
The energy of a photon is like the mind of God, It may very well exist.
Some people will tell you in detail  what they think it is.
And it's impossible to verify.
Title: Re: Where does quantization of energy of electromagnetic radiation come from?
Post by: alancalverd on 23/10/2023 15:32:10
But very easy to determine what it was before you measured it!
Title: Re: Where does quantization of energy of electromagnetic radiation come from?
Post by: hamdani yusuf on 23/10/2023 22:34:10
But very easy to determine what it was before you measured it!
How is it?
Title: Re: Where does quantization of energy of electromagnetic radiation come from?
Post by: hamdani yusuf on 23/10/2023 22:40:19
Thanks to indeterminacy you can't predict the energy of a given photon, nor the time at which it will arrive at a point,  with absolute accuracy, but as far as the photon is concerned it only has one energy.
Having one energy doesn't necessarily mean having one frequency. Mathematically, a single narrow frequency requires extremely long duration of sinusoidal wave.
Title: Re: Where does quantization of energy of electromagnetic radiation come from?
Post by: hamdani yusuf on 25/10/2023 13:14:29
But these are not enough to explain the sparkling effect.
I think it perfectly explains the effect. 
Do you now realize that the sparkling is not quantized?
We know for sure that electronic sensors are spatially quantized, although it isn't necessarily true that every pixel has the same size or sensitivity.
Do you think it has no observable effect?
This video shows the sparkling effect using a relatively low resolution camera sensor.

The presence of other effects, like diffraction, flaring, scattering, and various sizes of the ripple makes it hard to exclusively observe the sparkling effect.
Title: Re: Where does quantization of energy of electromagnetic radiation come from?
Post by: hamdani yusuf on 25/10/2023 13:26:02
In this video the quantization of the sparkling effect is most visible around time 0:50 to 1:00 mark, when it's zoomed in. It's unfortunate that the camera is out of focus which makes the bright spots blur.
Title: Re: Where does quantization of energy of electromagnetic radiation come from?
Post by: hamdani yusuf on 26/10/2023 01:10:42
If you are still convinced that there's no quantization effect whatsoever in the sparkling of rippling water surface when reflecting sunlight, I'm curious what does it take to make you change your mind?
Title: Re: Where does quantization of energy of electromagnetic radiation come from?
Post by: hamdani yusuf on 26/10/2023 06:24:11
The presence of other effects, like diffraction, flaring, scattering, and various sizes of the ripple makes it hard to exclusively observe the sparkling effect.
In modern digital cameras, including those in smart phones, software filters and image processing algorithms can also affect the results of sparkling effect.
The resolution of the video files are typically lower than the resolution of main camera sensor. In my video, the resolution is 0.92 megapixels (1280 x 720), although the main camera itself has resolution of 50 megapixels. Some image processing must be involved in reducing the resolution.
Title: Re: Where does quantization of energy of electromagnetic radiation come from?
Post by: hamdani yusuf on 26/10/2023 06:44:16

This short video shows the sparkling effect in slow motion.
Title: Re: Where does quantization of energy of electromagnetic radiation come from?
Post by: hamdani yusuf on 26/10/2023 08:24:43
I've recorded for a new video showing this sparkling effect. In one of the cut scenes, there's visible effect of diffraction caused by trace of wiped sweat on the camera glass. But the sparkling tiny bright spots don't seem to be diffracted.
Title: Re: Where does quantization of energy of electromagnetic radiation come from?
Post by: Bored chemist on 26/10/2023 16:30:56
If you are still convinced that there's no quantization effect whatsoever in the sparkling of rippling water surface when reflecting sunlight, I'm curious what does it take to make you change your mind?
Evidence.

But you probably need to start by saying what you think quantization means.
Title: Re: Where does quantization of energy of electromagnetic radiation come from?
Post by: alancalverd on 26/10/2023 22:35:23
Quote from: alancalverd on 23/10/2023 15:32:10
But very easy to determine what it was before you measured it!
How is it?
E = hν
Use a calorimeter or photocell to determine E. In theory, at least.
Title: Re: Where does quantization of energy of electromagnetic radiation come from?
Post by: Bored chemist on 26/10/2023 23:50:10
Use a calorimeter or photocell to determine E.
I'm sure I heard about someone doing that.
It gave the wrong answer.
I think it's because they forgot about chemistry.
Title: Re: Where does quantization of energy of electromagnetic radiation come from?
Post by: hamdani yusuf on 27/10/2023 04:58:38
I've recorded for a new video showing this sparkling effect. In one of the cut scenes, there's visible effect of diffraction caused by trace of wiped sweat on the camera glass. But the sparkling tiny bright spots don't seem to be diffracted.
Here are some of the screen shots.
(https://www.thenakedscientists.com/forum/index.php?action=dlattach;topic=81120.0;attach=34135)

(https://www.thenakedscientists.com/forum/index.php?action=dlattach;topic=81120.0;attach=34133)

The screenshots are 20 milliseconds apart, starting from the top left corner to the right, and ends up at the bottom right.
The images are taken from the same video. The top set are taken from different area than the bottom set, as you can see the difference in their brightness.
Title: Re: Where does quantization of energy of electromagnetic radiation come from?
Post by: hamdani yusuf on 27/10/2023 05:12:57
Evidence.
What kind of evidence do you need?
Can you be more specific?

Title: Re: Where does quantization of energy of electromagnetic radiation come from?
Post by: hamdani yusuf on 27/10/2023 05:28:14
But you probably need to start by saying what you think quantization means.
I asked Google Bard. I think it gives pretty good answer.
Quote
Quantization is the process of converting a continuous signal into a discrete signal. This is done by mapping the continuous signal to a finite set of values. Quantization is used in a variety of applications, including digital signal processing, image processing, and machine learning.

There are two main types of quantization:

Uniform quantization: This is the simplest type of quantization, where the continuous signal is divided into equal intervals and each interval is mapped to a unique value.
Non-uniform quantization: This type of quantization is used when the continuous signal has a non-uniform distribution. In this case, the intervals are sized differently to better represent the distribution of the signal.
Quantization can be a lossy process, meaning that some information is lost in the conversion from a continuous signal to a discrete signal. However, the amount of information lost can be controlled by the number of quantization levels used. More quantization levels will result in less information loss, but will also require more storage space.

Here are some examples of quantization:

When you take a digital photo, the camera quantizes the continuous light signal into a discrete set of pixel values.
When you encode an audio signal for MP3 playback, the audio signal is quantized to reduce the file size.
When you train a machine learning model, the model's weights are quantized to reduce the model's size and improve its performance on embedded devices.
Quantization is a powerful tool that can be used to improve the efficiency and performance of a variety of applications. However, it is important to understand the trade-offs involved in quantization before using it.

Digital cameras and human eyes have inherent structures which cause spatial quantization. They can't detect the image of an object that can't activate one pixel.
Title: Re: Where does quantization of energy of electromagnetic radiation come from?
Post by: hamdani yusuf on 27/10/2023 05:39:08
Here are some more screenshots.
(https://www.thenakedscientists.com/forum/index.php?action=dlattach;topic=81120.0;attach=34139)
This one shows the sparkling effect with various size.

(https://www.thenakedscientists.com/forum/index.php?action=dlattach;topic=81120.0;attach=34141)
This one shows the sparkling effect while also showing some diffraction effect. The small bright spots don't seem to show the diffraction effect.
Title: Re: Where does quantization of energy of electromagnetic radiation come from?
Post by: alancalverd on 27/10/2023 08:40:51
Use a calorimeter or photocell to determine E.
I'm sure I heard about someone doing that.
It gave the wrong answer.
I think it's because they forgot about chemistry.

Not entirely.

I built an x-ray calorimeter with graphite because it has about one sixth of the specific heat capacity of water (the target material for radiotherapy) and thus a larger temperature rise per unit dose, making the electronics fairly simple*. This is the UK primary measurement standard, and similar devices are used in all national standards labs, but oncologists would like to have a direct measure of dose to water rather than use theoretical conversion factors.

Then several labs built water calorimeters with more sensitive thermometry, and discovered that the discrepancy between water and graphite measurements was (a) greater than the error range of the conversion factors and (b) decreased at very high continuous doses, so AFAIK (I haven't worked on calorimetry for several years) the decision was to stick with graphite and physics rather than try to understand and circumvent the chemistry.

*we needed to measure 0.001K temperature rise to better than +/- 0.000001K. I guess it's an idiosyncratic interpretation of "simple", but it was still easier than resolving 0.0002K to the same  uncertainty.   
Title: Re: Where does quantization of energy of electromagnetic radiation come from?
Post by: hamdani yusuf on 27/10/2023 12:58:57
Then several labs built water calorimeters
What was the expected advantage of water calorimeters over the graphite?
Title: Re: Where does quantization of energy of electromagnetic radiation come from?
Post by: hamdani yusuf on 28/10/2023 11:42:45
There are several modes of sparkling effect that can be observed from the videos. But basically, it's caused by reflection of bright light source by rippling surface of water, which produces several images of that light source on the surface of photosensitive detector. Depending on the curvature of the ripples, and the distance between the reflecting surface and the detectors, the size of the image formed on the detector may vary.
The image of an object reflected by a convex surface is smaller than the same image of the same object reflected by a flat surface. A concave surface may produce larger image at certain distance, but for significantly larger distance than the radius of the curvature, it also produces smaller image. At large enough distance, the image can be even smaller than the size of a single pixel in the photosensor. But since the light source is excessively bright, it can still saturate that photosensor pixel.
These diagrams show some of the modes. Each grey rectangle represents a pixel of a photosensor. The first may explain how a single bright spot can be seen steadily lit up for several frames until it suddenly turn off.

(https://www.thenakedscientists.com/forum/index.php?action=dlattach;topic=81120.0;attach=34145)

(https://www.thenakedscientists.com/forum/index.php?action=dlattach;topic=81120.0;attach=34147)
Title: Re: Where does quantization of energy of electromagnetic radiation come from?
Post by: hamdani yusuf on 28/10/2023 13:34:15
Image size and intensity also affect the recorded image on the sensor.
Title: Re: Where does quantization of energy of electromagnetic radiation come from?
Post by: hamdani yusuf on 28/10/2023 13:35:42
Finally, here's how diffraction effect doesn't seem to affect small sparkling spots.
(https://www.thenakedscientists.com/forum/index.php?action=dlattach;topic=81120.0;attach=34155)
Title: Re: Where does quantization of energy of electromagnetic radiation come from?
Post by: alancalverd on 28/10/2023 23:49:54
Then several labs built water calorimeters
What was the expected advantage of water calorimeters over the graphite?
The effect of radiation on living tissue is principally due to its interaction with cellular cytoplasm - mostly water. So for radiotherapy, in particular, we would like to know the dose (amount of energy deposited per unit mass) delivered to water by our radiation beam. We can make a theoretical estimate from the electron density of graphite but it would have been nice to eliminate all the approximations inherent in that calculation.

I did some work with very large initial doses and doserates, so that the water had reached something of a chemical equilibrium before the actual measurement phase, but the results were still too variable to be clinically useful. Pity, because the apparatus was delightfully simple - just a bucket and a thermistor!
Title: Re: Where does quantization of energy of electromagnetic radiation come from?
Post by: alancalverd on 28/10/2023 23:51:41
Your pixels seem to be binary, whereas real camera pixels are linear. 
Title: Re: Where does quantization of energy of electromagnetic radiation come from?
Post by: hamdani yusuf on 29/10/2023 06:31:05

Your pixels seem to be binary, whereas real camera pixels are linear. 
I think you missed to see my previous post.
Image size and intensity also affect the recorded image on the sensor.
(https://www.thenakedscientists.com/forum/index.php?action=dlattach;topic=81120.0;attach=34151)
Title: Re: Where does quantization of energy of electromagnetic radiation come from?
Post by: hamdani yusuf on 29/10/2023 06:49:44
The pictures I posted here are screenshots of my next videos. The first will show a closer look to the phenomenon, while the second one tries to offer some explanations.
The existence of several effects at once in a single piece of evidence may prevent us from identifying the phenomenon in the first place.

I'd like to have some constructive feedback to improve my videos before uploading them. Does anyone notice some unusual effects I haven't mentioned yet? Or think that the effects I did mention here implausible?
Title: Re: Where does quantization of energy of electromagnetic radiation come from?
Post by: hamdani yusuf on 30/10/2023 15:02:09
Here it is. A closer look at sparkling water surface.
Title: Re: Where does quantization of energy of electromagnetic radiation come from?
Post by: alancalverd on 30/10/2023 17:06:36
Everything you see is entirely plausible. Not every explanatory hypothesis is plausible. That's science!
Title: Re: Where does quantization of energy of electromagnetic radiation come from?
Post by: Bored chemist on 31/10/2023 09:42:24
Here it is. A closer look at sparkling water surface.

What needs explaining?
Title: Re: Where does quantization of energy of electromagnetic radiation come from?
Post by: hamdani yusuf on 01/11/2023 13:35:18
Everything you see is entirely plausible. Not every explanatory hypothesis is plausible. That's science!
Do you have an explanatory hypothesis for the observation?
Title: Re: Where does quantization of energy of electromagnetic radiation come from?
Post by: hamdani yusuf on 01/11/2023 13:54:21
What needs explaining?
Anything that's seem unusual from a typical image reflected by water surface.
Title: Re: Where does quantization of energy of electromagnetic radiation come from?
Post by: Origin on 01/11/2023 14:03:41
Anything that's seem unusual from a typical image reflected by water surface.
Nothing is unusual, so no explanation is necessary.  Easy-peasy.
Title: Re: Where does quantization of energy of electromagnetic radiation come from?
Post by: Bored chemist on 01/11/2023 14:10:04
Do you have an explanatory hypothesis for the observation?
Water reflect light.
The surface of water is disturbed by things like the wind and fountains.

What else needs explaining?
Title: Re: Where does quantization of energy of electromagnetic radiation come from?
Post by: Bored chemist on 01/11/2023 14:11:01
Anything that's seem unusual
What do you think is unusual?
Everyone else here seem to think that what you see is exactly what we would expect to see.
Title: Re: Where does quantization of energy of electromagnetic radiation come from?
Post by: hamdani yusuf on 02/11/2023 07:24:27
Do you have an explanatory hypothesis for the observation?
Water reflect light.
The surface of water is disturbed by things like the wind and fountains.

What else needs explaining?
Anything that's seem unusual
What do you think is unusual?
Everyone else here seem to think that what you see is exactly what we would expect to see.
I've identified 6 effects related to saturation and pixelation/spatial quantization of the photosensors. What do you think would happen if those effects don't actually occur? 
Title: Re: Where does quantization of energy of electromagnetic radiation come from?
Post by: paul cotter on 02/11/2023 08:47:47
Quantisation in analogue to digital converters and quantisation of photon energy are entirely separate phenomena.
Title: Re: Where does quantization of energy of electromagnetic radiation come from?
Post by: hamdani yusuf on 02/11/2023 09:50:45
Quantisation in analogue to digital converters and quantisation of photon energy are entirely separate phenomena.
What makes them different?
Title: Re: Where does quantization of energy of electromagnetic radiation come from?
Post by: Bored chemist on 02/11/2023 11:16:30
I've identified 6 effects related to saturation and pixelation/spatial quantization of the photosensors.
Well, I'm not convinced about the nomenclature, but the effects seem reasonable.
So what?
They are not "unusual".

What do you think would happen if those effects don't actually occur? 
Since they do, in fact, occur, why should I concern myself ?
It's like asking what would happen if it rained cows.
Title: Re: Where does quantization of energy of electromagnetic radiation come from?
Post by: hamdani yusuf on 02/11/2023 13:41:03
I've identified 6 effects related to saturation and pixelation/spatial quantization of the photosensors.
Well, I'm not convinced about the nomenclature, but the effects seem reasonable.
So what?
They are not "unusual".

What do you think would happen if those effects don't actually occur?
Since they do, in fact, occur, why should I concern myself ?
It's like asking what would happen if it rained cows.


That's good if you think that the effects are reasonable. Do you find other effects not yet in my list?
It's understandable if you are not interested in investigating the phenomena of sparkling water. Some people don't even care about basic sciences if they don't directly affect their lives. I posted here for those who share my curiosity of the phenomenon I've observed, which doesn't seem to be fully explained by simple reflection from a randomly rippling reflective surface.

I'd like to share an educational video on constructing scientific explanations.
Constructing Explanations - Level 1 - Observational Explanations
Quote
In this video Paul Andersen shows you how to construct explanations with evidence in a mini-lesson on Observational Explanations.  Two examples are included in the video and two additional examples are included in the linked thinking slides. 

TERMS
Explanation - a logical reason for a phenomenon
Observations - a statement about something you notice
Phenomena - observable events in the natural world (require explanations)
Question - a sentence that asks for information
Reasoning - the action of thinking about something in a logical way

This progression is based on the Science and Engineering Practices elements from the NRC document A Framework for K-12 Science Education.  ?Make observations (firsthand or from media) to construct an evidence-based account for natural phenomena.?
Source:  https://www.nextgenscience.org/
Title: Re: Where does quantization of energy of electromagnetic radiation come from?
Post by: alancalverd on 02/11/2023 17:39:34
Quantisation in analogue to digital converters and quantisation of photon energy are entirely separate phenomena.
What makes them different?
The word has two different meanings (a) representing a continuous function with a discontinuous function and (b) modelling the behavior of electromagnetic radiation as a stream of particles each having a discrete quantity of energy.
Title: Re: Where does quantization of energy of electromagnetic radiation come from?
Post by: Bored chemist on 02/11/2023 18:29:31
which doesn't seem to be fully explained by simple reflection from a randomly rippling reflective surface.
And again....
what bits do you think are not explained?
Title: Re: Where does quantization of energy of electromagnetic radiation come from?
Post by: hamdani yusuf on 03/11/2023 02:21:03
which doesn't seem to be fully explained by simple reflection from a randomly rippling reflective surface.
And again....
what bits do you think are not explained?
They won't be explained if the pixelation and saturation of the photosensors were excluded.
Title: Re: Where does quantization of energy of electromagnetic radiation come from?
Post by: hamdani yusuf on 03/11/2023 02:46:28
Do you find other effects not yet in my list?

There is another effect I can identify, although it's better visualized in other video.
(https://www.thenakedscientists.com/forum/index.php?action=dlattach;topic=81120.0;attach=34157)
In the case where the size of the ripples are small enough or the distance to the sensor is far enough, there will be many small images of the sun in each pixel. The intensity may not be enough to cause pixel saturation. But the spreading out of the reflection makes the overall image size of the sun look larger than reflection from a completely smooth surface at the same distance.
This one shows Sun Reflection on Pacific Ocean from Space.
Title: Re: Where does quantization of energy of electromagnetic radiation come from?
Post by: Bored chemist on 03/11/2023 12:02:11
They won't be explained if the pixelation and saturation of the photosensors were excluded.

Why would you exclude pixelation and saturation, knowing that doing so would give you the wrong answer
Title: Re: Where does quantization of energy of electromagnetic radiation come from?
Post by: hamdani yusuf on 03/11/2023 12:40:16
They won't be explained if the pixelation and saturation of the photosensors were excluded.

Why would you exclude pixelation and saturation, knowing that doing so would give you the wrong answer
Because someone said they are not necessary.

But these are not enough to explain the sparkling effect.
I think it perfectly explains the effect. 
Do you now realize that the sparkling is not quantized?

Anything that's seem unusual from a typical image reflected by water surface.
Nothing is unusual, so no explanation is necessary.  Easy-peasy.


Do you have an explanatory hypothesis for the observation?
Water reflect light.
The surface of water is disturbed by things like the wind and fountains.

What else needs explaining?
Title: Re: Where does quantization of energy of electromagnetic radiation come from?
Post by: alancalverd on 03/11/2023 15:41:11
Just to add mischief to your confusion, remember that the displayed image is not usually a 1:1 mapping of the receptor pixel signals. Right now I am using a 3840 x 2160 display - about 8 Mpx - but my camera has a 16 Mpx receptor and the webcam on the computer is only 500k. There are all sorts of software interventions that "hide the joins" and either smooth out or enhance contrast in adjacent areas. In radiology we tend to capture images with the highest available spatial resolution but often "soften" the display so that tiny "punctate" features are more conspicuous - there is an optimum contrast/detail balance somewhere between the maxima  of either variable.
Title: Re: Where does quantization of energy of electromagnetic radiation come from?
Post by: hamdani yusuf on 04/11/2023 08:05:58
Just to add mischief to your confusion, remember that the displayed image is not usually a 1:1 mapping of the receptor pixel signals. Right now I am using a 3840 x 2160 display - about 8 Mpx - but my camera has a 16 Mpx receptor and the webcam on the computer is only 500k. There are all sorts of software interventions that "hide the joins" and either smooth out or enhance contrast in adjacent areas. In radiology we tend to capture images with the highest available spatial resolution but often "soften" the display so that tiny "punctate" features are more conspicuous - there is an optimum contrast/detail balance somewhere between the maxima  of either variable.
I'm aware of your concerns here.
The presence of other effects, like diffraction, flaring, scattering, and various sizes of the ripple makes it hard to exclusively observe the sparkling effect.
In modern digital cameras, including those in smart phones, software filters and image processing algorithms can also affect the results of sparkling effect.
The resolution of the video files are typically lower than the resolution of main camera sensor. In my video, the resolution is 0.92 megapixels (1280 x 720), although the main camera itself has resolution of 50 megapixels. Some image processing must be involved in reducing the resolution.
That's why I posted many videos from other Youtubers to show variations of camera resolution, their contrast and saturation levels, and resolution of the resulting videos.
Title: Re: Where does quantization of energy of electromagnetic radiation come from?
Post by: hamdani yusuf on 04/11/2023 08:19:33
Interference pattern built up photon by photon
Quote
This movie has been captured with an intensified CCD camera. The movie consists of 200 frames, with exposure times ranging between 0,025 milliseconds and 6,000 milliseconds. It shows how individual photons, transmitted through a double slit, form an interference pattern. It shows wave-particle duality of light.
Does anyone notice that the bright spots have various brightness? How should we interpret it?

Moreover, what is the size of the photons producing that bright spots?
Do they depend on their frequency?
Do they depend on their polarization?
Title: Re: Where does quantization of energy of electromagnetic radiation come from?
Post by: alancalverd on 04/11/2023 08:54:05
That's why I posted many videos from other Youtubers to show variations of camera resolution, their contrast and saturation levels, and resolution of the resulting videos.

None of which relates quantisation of photon energy to the appearance of sunlight reflected from rippling water.
Title: Re: Where does quantization of energy of electromagnetic radiation come from?
Post by: hamdani yusuf on 04/11/2023 08:56:54
For comparison, double slit experiment using electrons shows more uniform intensity in each bright spot, at least until one spot is hit more than once.

Single electron double slit wave experiment
Title: Re: Where does quantization of energy of electromagnetic radiation come from?
Post by: hamdani yusuf on 04/11/2023 08:58:07
That's why I posted many videos from other Youtubers to show variations of camera resolution, their contrast and saturation levels, and resolution of the resulting videos.

None of which relates quantisation of photon energy to the appearance of sunlight reflected from rippling water.
The relationship lies in how the photosensitive sensors work.
Title: Re: Where does quantization of energy of electromagnetic radiation come from?
Post by: alancalverd on 04/11/2023 09:09:02
Does anyone notice that the bright spots have various brightness? How should we interpret it?
The signal delivered by a single pixel irradiated by a single photon depends on the photon energy (which we can assume constant in this instance as a fairly good interference pattern appears) and the time between impact and readout (which is random). 

In this case they talk about an "intensified" CCD, which adds another time delay and decay curve. Not sure what intensifier they are using here, but x-ray intensifying screens generate multiple visible photons from the impact of a single x-ray photon, with a pronounced intensity decay curve with a time constant ranging from nanoseconds (for fast movie work) to minutes (useful for engineers to align x-ray systems visually). So the delivered image brightness associated with a single incident photon doesn't have much to do with the originating photon but a lot to do with the detector mechainkism.
Title: Re: Where does quantization of energy of electromagnetic radiation come from?
Post by: hamdani yusuf on 04/11/2023 09:10:09
This is a reminder that when something weird or illogical or unexpected happens, it's because we've made one or more false assumptions.


Boy, Was I Wrong! How the Delayed Choice Quantum Eraser Really works

Quote
CHAPTERS
0:00 The original paper implied retrocausality
1:23 Really cool metal posters: Displates!
2:37 A classical interpretation would show retrocausality
3:49 How the double slit experiment works
6:25 Debunking the clean double line pattern
7:49 The Delayed Choice Quantum Eraser set up explained
11:54 How the Scientis hand-selected the outcome of the Delayed Choice experiment

SUMMARY
The original paper by the authors who first performed the Delayed Choice Quantum Eraser implied retro causality. But retro causality is true only if you assume a classical way of thinking. But that's not the way quantum mechanics works, and I was wrong for interpreting it that way in my original 2019 video. When viewed with the standard interpretation of quantum mechanics where a particle is always a wave until the moment it is measured, there is no retro causality.

How the double slit experiment works: If you send photons one at a time through the slits, at first you will see what looks like a random distribution of dots. But after a while, you will see that those dots create an interference pattern.

If you then put detectors on the slits to measure which slit the photon passes through, you see a pattern like you would if you were sending individual particles through the slits. The act of measuring seems to affect the results. But the change is due to the nature of quantum mechanics. All quantum objects like photons and electrons are really waves. But if they interact with anything, that is, if an irreversible energy exchange takes place, their waves become localized like a particle. This is called ?wave collapse.? Wave collapse also occurs when the photon interacts with the screen in the back. And we this as a dot on the screen.

The Delayed Choice Quantum Eraser is like the double slit experiment on steroids. First, I want to point out that if you have a detector that measures the path, you don?t really get two clean lines of photons like it's usually illustrated. You get a single spread out distribution of photons.

How does the delayed choice experiment work?
It starts with the double slit, but first the photons go through a special optical device called a Barium Borate crystal. It splits a single photon into a pair of entangled photons with half the energy each of the original. Note that the process of creating entangled photons effectively results in a measurement. In other words, the wave function of the photon collapses so that it is now a particle. And since the path from the top slit to detector 1 is slightly different than the path from detector 2, the which way information of the photon is known. Thus the pattern that will show up at detector 1 will always be a spread out pattern, not an interference pattern. It doesn?t matter what happens at any of the other detectors.

So why is it illustrated as changing depending on what happens at the other detectors? This is the center of the confusion, and where the idea of retro causality comes in.

Well the confusion is from the way this experiment is presented - as D1 changing its pattern to match the interference pattern at D4 or D5 when the photons end up there, but showing a different pattern, a spread out pattern, if the photons end up at D2 or D3.

So this implies that what happens at D2, D3, D4 or D5 influences what happens at D1. But since the path to D1 is shorter than the path to any of the other detectors, the photons reach D1 BEFORE they reach D2, D3, D4 or D5. So the implication is that the pattern at D1 which would be in the past, is being affected by what happens in the future at D2, D3, D4 or D5. So people have naturally been led to think that this means retro causality. This is wrong.

The quantum eraser has no effect on the original screen. What?s really happening is that the changing patterns are due to the scientists, conducting this experiment, selecting subsets of the photons in D1 to show the same patterns as at each of the other detectors. This can be done because the particles hitting the screen at D1 and the particles going to the other detectors are entangled.
#delayedchoicequantumeraser
#quantumphysics
So in the presentations that you see, including the one I originally made, the interference pattern you see get at D1 is nothing but a hand-selected subset of the actual original spread out pattern at D1, corresponding to photons that ended up at D4 or D5. This is done post-experiment by hand! The patterns do not change on their own. The future does not affect the past.
Title: Re: Where does quantization of energy of electromagnetic radiation come from?
Post by: alancalverd on 04/11/2023 09:11:47
For comparison, double slit experiment using electrons shows more uniform intensity in each bright spot, at least until one spot is hit more than once.
Because the electron charge is a constant, unlike the signal you get from an intensified photon detector.
Title: Re: Where does quantization of energy of electromagnetic radiation come from?
Post by: hamdani yusuf on 04/11/2023 09:13:32
Interference pattern built up photon by photon
Quote
This movie has been captured with an intensified CCD camera. The movie consists of 200 frames, with exposure times ranging between 0,025 milliseconds and 6,000 milliseconds. It shows how individual photons, transmitted through a double slit, form an interference pattern. It shows wave-particle duality of light.
Does anyone notice that the bright spots have various brightness? How should we interpret it?

Moreover, what is the size of the photons producing that bright spots?
Do they depend on their frequency?
Do they depend on their polarization?

While in this video, the difference in size of the bright spots seems to be more visible.
Single Photon Interference Double Slit
Title: Re: Where does quantization of energy of electromagnetic radiation come from?
Post by: hamdani yusuf on 04/11/2023 09:27:11
Boy, Was I Wrong! How the Delayed Choice Quantum Eraser Really works
It shows The Problem With Science Communication.
Title: Re: Where does quantization of energy of electromagnetic radiation come from?
Post by: alancalverd on 04/11/2023 10:48:27
Moreover, what is the size of the photons producing that bright spots?
Do they depend on their frequency?
Do they depend on their polarization?

Not sure what you mean by "size" of a photon. SInce there is a clear interference pattern. the photons will all have had pretty much the same frequency/wavelength/energy.
Title: Re: Where does quantization of energy of electromagnetic radiation come from?
Post by: hamdani yusuf on 04/11/2023 11:29:53
Moreover, what is the size of the photons producing that bright spots?
Do they depend on their frequency?
Do they depend on their polarization?

Not sure what you mean by "size" of a photon. SInce there is a clear interference pattern. the photons will all have had pretty much the same frequency/wavelength/energy.
What determined the diameter of those bright spots on the screen in those single photon double slit experiments?
If the frequency of the light source is doubled, would it change the diameter of those bright spots?
Title: Re: Where does quantization of energy of electromagnetic radiation come from?
Post by: alancalverd on 05/11/2023 10:34:36
What determined the diameter of those bright spots on the screen in those single photon double slit experiments?
See reply #257 above.
If the frequency of the light source is doubled, would it change the diameter of those bright spots?
Generally, yes, because each interaction with the primary detector will release twice as much energy, either in the form of visible photons or electrons depending on the incident radiation and the  type of detector used. Most likely electrons.
Title: Re: Where does quantization of energy of electromagnetic radiation come from?
Post by: hamdani yusuf on 06/11/2023 13:08:09
The pictures I posted here are screenshots of my next videos. The first will show a closer look to the phenomenon, while the second one tries to offer some explanations.
The existence of several effects at once in a single piece of evidence may prevent us from identifying the phenomenon in the first place.

I'd like to have some constructive feedback to improve my videos before uploading them. Does anyone notice some unusual effects I haven't mentioned yet? Or think that the effects I did mention here implausible?
Here's the video trying to explain the effects found in sparkling water surface videos.

Title: Re: Where does quantization of energy of electromagnetic radiation come from?
Post by: hamdani yusuf on 06/11/2023 13:15:20
What determined the diameter of those bright spots on the screen in those single photon double slit experiments?
See reply #257 above.
If the frequency of the light source is doubled, would it change the diameter of those bright spots?
Generally, yes, because each interaction with the primary detector will release twice as much energy, either in the form of visible photons or electrons depending on the incident radiation and the  type of detector used. Most likely electrons.
Quote
https://www.tedpella.com/cameras_html/ccd_cmos.aspx

(https://www.tedpella.com/cameras_html/ccd.jpg)
Figure 1: Diagram of a CCD.
On a CCD, most functions take place on the camera's printed circuit board. If the application's demands change, a designer can change the electronics without redesigning the imager.

(https://www.tedpella.com/cameras_html/cmos.jpg)
Figure 2: Diagram of a CMOS.
A CMOS imager converts charge to voltage at the pixel, and most functions are integrated into the chip. This makes imager functions less flexible but, for applications in rugged environments, a CMOS camera can be more reliable.


This difference in readout techniques has significant implications for sensor capabilities and limitations. Eight attributes characterize image sensor performance.

Responsivity, the amount of signal the sensor delivers per unit of input optical energy. CMOS imagers are marginally superior to CCDs.
Dynamic range, the ratio of a pixel's saturation level to its signal threshold. CCDs have the advantage here.
Uniformity, is the consistency of response for different pixels under identical illumination conditions. CMOS imagers were traditionally much worse than CCDs, however new amplifiers have made the illuminated uniformity of some CMOS imagers close to that of CCDs.
Shuttering, the ability to start and stop exposure arbitrarly, is superior in CCD devices. CMOS devices require extra transistors or nonuniform shuttering, sometimes called rolling shuttering to achieve the same results.
Speed, an area in which CMOS arguably has the advantage over CCDs because all of the camera functions can be placed on the image sensor.
Windowing, CMOS technology has the ability to read out a portion of the image sensor allowing elevated frame rates for small regions of interest. CCDs generally have limited abilities in windowing.
Antiblooming, is the ability to gracefully drain localized overexposure without compromising the rest of the image in the sensor. CMOS generally has natural blooming immunity. CCDs require specific engineering to achieve this capability.
Biasing and clocking. CMOS imagers have a clear advantage in the area, operating on a single bias voltage and clock level.

CCD and CMOS imagers were both invented in the late 1960's. CCD became dominant in the market, primarily because they produced superior images with the fabrication technology available. CMOS image sensors required more uniformity and smaller features than silicon wafer foundries could deliver at the time. Not until the 1990's, with the development of lithography was there a renewed interest in CMOS. That interest is due to lower power consumption, camera-on-a-chip integration, and lowered fabrication costs. Both CCD and CMOS imagers offer excellent imaging performance. CMOS imagers offer more integration (more functions on the chip), lower power dissipation (at the chip level), and the possibility of smaller system size.

Today there is no clear line dividing the types of applications each can serve. CCD and CMOS technologies are used interchangeably. CMOS designers have devoted intense effort to achieving high image quality, while CCD designers have lowered their power requirements and their pixel sizes. As a result, you can find CMOS sensors in high-performance professional and industrial cameras and CCDs in low cost low power cell phone cameras. For the moment, CCDs and CMOS remain complementary technologies-one can do things uniquely the other cannot. Over time this distinction will soften, with more CMOS imagers consuming more and more of the CCD's traditional applications. Considering the relative strength and opportunities of CCD and CMOS imagers, the choice continues to depend on the application and the vendor more than the technology.
Title: Re: Where does quantization of energy of electromagnetic radiation come from?
Post by: hamdani yusuf on 06/11/2023 13:17:20
What determined the diameter of those bright spots on the screen in those single photon double slit experiments?
See reply #257 above.
If the frequency of the light source is doubled, would it change the diameter of those bright spots?
Generally, yes, because each interaction with the primary detector will release twice as much energy, either in the form of visible photons or electrons depending on the incident radiation and the  type of detector used. Most likely electrons.

Here's how photosensors work. How do you think the double in size of the bright spots come out?
Quote
https://www.tedpella.com/cameras_html/ccd_cmos.aspx

(https://www.tedpella.com/cameras_html/ccd.jpg)
Figure 1: Diagram of a CCD.
On a CCD, most functions take place on the camera's printed circuit board. If the application's demands change, a designer can change the electronics without redesigning the imager.

(https://www.tedpella.com/cameras_html/cmos.jpg)
Figure 2: Diagram of a CMOS.
A CMOS imager converts charge to voltage at the pixel, and most functions are integrated into the chip. This makes imager functions less flexible but, for applications in rugged environments, a CMOS camera can be more reliable.


This difference in readout techniques has significant implications for sensor capabilities and limitations. Eight attributes characterize image sensor performance.

Responsivity, the amount of signal the sensor delivers per unit of input optical energy. CMOS imagers are marginally superior to CCDs.
Dynamic range, the ratio of a pixel's saturation level to its signal threshold. CCDs have the advantage here.
Uniformity, is the consistency of response for different pixels under identical illumination conditions. CMOS imagers were traditionally much worse than CCDs, however new amplifiers have made the illuminated uniformity of some CMOS imagers close to that of CCDs.
Shuttering, the ability to start and stop exposure arbitrarly, is superior in CCD devices. CMOS devices require extra transistors or nonuniform shuttering, sometimes called rolling shuttering to achieve the same results.
Speed, an area in which CMOS arguably has the advantage over CCDs because all of the camera functions can be placed on the image sensor.
Windowing, CMOS technology has the ability to read out a portion of the image sensor allowing elevated frame rates for small regions of interest. CCDs generally have limited abilities in windowing.
Antiblooming, is the ability to gracefully drain localized overexposure without compromising the rest of the image in the sensor. CMOS generally has natural blooming immunity. CCDs require specific engineering to achieve this capability.
Biasing and clocking. CMOS imagers have a clear advantage in the area, operating on a single bias voltage and clock level.

CCD and CMOS imagers were both invented in the late 1960's. CCD became dominant in the market, primarily because they produced superior images with the fabrication technology available. CMOS image sensors required more uniformity and smaller features than silicon wafer foundries could deliver at the time. Not until the 1990's, with the development of lithography was there a renewed interest in CMOS. That interest is due to lower power consumption, camera-on-a-chip integration, and lowered fabrication costs. Both CCD and CMOS imagers offer excellent imaging performance. CMOS imagers offer more integration (more functions on the chip), lower power dissipation (at the chip level), and the possibility of smaller system size.

Today there is no clear line dividing the types of applications each can serve. CCD and CMOS technologies are used interchangeably. CMOS designers have devoted intense effort to achieving high image quality, while CCD designers have lowered their power requirements and their pixel sizes. As a result, you can find CMOS sensors in high-performance professional and industrial cameras and CCDs in low cost low power cell phone cameras. For the moment, CCDs and CMOS remain complementary technologies-one can do things uniquely the other cannot. Over time this distinction will soften, with more CMOS imagers consuming more and more of the CCD's traditional applications. Considering the relative strength and opportunities of CCD and CMOS imagers, the choice continues to depend on the application and the vendor more than the technology.

Title: Re: Where does quantization of energy of electromagnetic radiation come from?
Post by: alancalverd on 06/11/2023 22:51:39
How do you think the double in size of the bright spots come out?
Scatter within the CCD, scatter within the intensifier, and halo from the optics, are the usual causes.
Title: Re: Where does quantization of energy of electromagnetic radiation come from?
Post by: hamdani yusuf on 07/11/2023 08:15:02
How do you think the double in size of the bright spots come out?
Scatter within the CCD, scatter within the intensifier, and halo from the optics, are the usual causes.
I was asking about doubling in size of the bright spot shown in resulting image when the frequency of light source is doubled, without changing anything else.
Title: Re: Where does quantization of energy of electromagnetic radiation come from?
Post by: alancalverd on 07/11/2023 22:09:46
See reply #264, second part.
Title: Re: Where does quantization of energy of electromagnetic radiation come from?
Post by: hamdani yusuf on 08/11/2023 13:27:28
See reply #264, second part.
Do you realize that pixel size is fixed?
Title: Re: Where does quantization of energy of electromagnetic radiation come from?
Post by: alancalverd on 08/11/2023 18:59:47
Yes, and the bright spots are larger than the pixels.
Title: Re: Where does quantization of energy of electromagnetic radiation come from?
Post by: hamdani yusuf on 09/11/2023 13:57:47
Yes, and the bright spots are larger than the pixels.
Activating two pixels at once will make the bright spot elongated.

What would happen if the photon frequency is only 50% higher than before?
Title: Re: Where does quantization of energy of electromagnetic radiation come from?
Post by: alancalverd on 09/11/2023 17:35:10
 
Activating two pixels at once will make the bright spot elongated.
The spots you can see on the image almost certainly span more than two pixels.

Quote
What would happen if the photon frequency is only 50% higher than before?
You get 50% more energy deposited in the first interaction.

The only system that is likely to register a single photon as a single "pixel" (note the inverted commas) is a single-emulsion x-ray film, where a single emulsion grain can be activated by one photon. Problem is that this is extremely insensitive, it's very difficult to register single-photon interference patterns at x-ray wavelengths, and you need a very good microscope to see the individual blackened grains. All practical single-photon detector systems use some form of  converter to produce lots of photons or lots of electrons from a single primary interaction, so the image you see isn't that of a single photon after all! 
Title: Re: Where does quantization of energy of electromagnetic radiation come from?
Post by: hamdani yusuf on 10/11/2023 09:20:43
The spots you can see on the image almost certainly span more than two pixels.
What happens in case of less than two pixels?

You get 50% more energy deposited in the first interaction.
In what form?
Title: Re: Where does quantization of energy of electromagnetic radiation come from?
Post by: hamdani yusuf on 11/11/2023 12:27:29
Quote
In quantum mechanics, wave function collapse occurs when a wave function?initially in a superposition of several eigenstates?reduces to a single eigenstate due to interaction with the external world.
https://en.m.wikipedia.org/wiki/Wave_function_collapse
In various experiments using dim light source, the wave function doesn't seem to collapse when it interacts with many kinds of objects such as mirrors, lens, polarizers, slits, gratings, quarter wave plates, half wave plates, air molecules, and beam splitters. Only certain kinds of objects can cause the wave function to collapse, such as electronic photosensors. There must be something that causes that difference in behavior.
Title: Re: Where does quantization of energy of electromagnetic radiation come from?
Post by: alancalverd on 11/11/2023 14:14:14
The spots you can see on the image almost certainly span more than two pixels.
What happens in case of less than two pixels?
You get a very tiny dot in the display

Quote
You get 50% more energy deposited in the first interaction.
In what form?
In whatever form the sensor converts it to. Could be ion formation, photoelectron emission, visible photons, or electrons promoted to a higher trap level in a phosphor.
Title: Re: Where does quantization of energy of electromagnetic radiation come from?
Post by: hamdani yusuf on 11/11/2023 16:21:47
You get a very tiny dot in the display
One pixel is the minimum non-zero result.
Title: Re: Where does quantization of energy of electromagnetic radiation come from?
Post by: hamdani yusuf on 22/11/2023 13:13:23
Quote
In quantum mechanics, wave function collapse occurs when a wave function?initially in a superposition of several eigenstates?reduces to a single eigenstate due to interaction with the external world.
https://en.m.wikipedia.org/wiki/Wave_function_collapse
In various experiments using dim light source, the wave function doesn't seem to collapse when it interacts with many kinds of objects such as mirrors, lens, polarizers, slits, gratings, quarter wave plates, half wave plates, air molecules, and beam splitters. Only certain kinds of objects can cause the wave function to collapse, such as electronic photosensors. There must be something that causes that difference in behavior.

In other part of the article, it says.
Quote
History and context
The concept of wavefunction collapse was introduced by Werner Heisenberg in his 1927 paper on the uncertainty principle, "?ber den anschaulichen Inhalt der quantentheoretischen Kinematik und Mechanik", and incorporated into the mathematical formulation of quantum mechanics by John von Neumann, in his 1932 treatise Mathematische Grundlagen der Quantenmechanik.[10] Heisenberg did not try to specify exactly what the collapse of the wavefunction meant. However, he emphasized that it should not be understood as a physical process.[11] Niels Bohr also repeatedly cautioned that we must give up a "pictorial representation", and perhaps also interpreted collapse as a formal, not physical, process.[12]

Consistent with Heisenberg, von Neumann postulated that there were two processes of wave function change:

The probabilistic, non-unitary, non-local, discontinuous change brought about by observation and measurement, as outlined above.
The deterministic, unitary, continuous time evolution of an isolated system that obeys the Schr?dinger equation (or a relativistic equivalent, i.e. the Dirac equation).
In general, quantum systems exist in superpositions of those basis states that most closely correspond to classical descriptions, and, in the absence of measurement, evolve according to the Schr?dinger equation. However, when a measurement is made, the wave function collapses?from an observer's perspective?to just one of the basis states, and the property being measured uniquely acquires the eigenvalue of that particular state,
λ. After the collapse, the system again evolves according to the Schr?dinger equation.
Title: Re: Where does quantization of energy of electromagnetic radiation come from?
Post by: hamdani yusuf on 26/11/2023 05:32:48
Intensity of a spherical light source is quartered when the distance is doubled. This and many other experiments show that light always propagate like a continuous wave, until it's detected. Which means it interact with some material objects non-linearly. That's what's called wave function collapse, IMO.
This non-linearity reduces information contained in the known system, just like saturation in the electronic components.
Title: Re: Where does quantization of energy of electromagnetic radiation come from?
Post by: hamdani yusuf on 13/02/2024 13:11:28
Quote
Confining the light causes it to have more colours. This is explained well in an excellent video by Ben Miles. But at first glance, the experiment seems to be a totally different one than what I explained in the video- so I'll explain what the connection is. In particular, it's only the "single slit" version of the experiment that's relevant for us. I said that the researchers confined a laser to a small space. The technique they used to do this was to have two lasers- one which is the source, and the other which is used to turn on and off a "switch" of sorts. What the switch does is it makes the material in the experiment go from transparent to reflective very quickly, then back. The source laser is shining continuously at the material.

But the idea is that for the short while that the material is reflective a little section of the laser beam is reflected. That's the "confined" light- they took a laser beam that's always on and constant and isolated a small section, confining the whereabouts of the light. They then measured the colours of that light and find it's spread out. (This result is at the 8 minute mark)
Title: Re: Where does quantization of energy of electromagnetic radiation come from?
Post by: alancalverd on 13/02/2024 22:15:06
Clearly written by someone with no understanding of physics.
Title: Re: Where does quantization of energy of electromagnetic radiation come from?
Post by: hamdani yusuf on 14/02/2024 01:45:44
Clearly written by someone with no understanding of physics.
Perhaps you just understand it differently than she does. At least her PhD advisors and reviewers agreed that she understand some of it.
Title: Re: Where does quantization of energy of electromagnetic radiation come from?
Post by: hamdani yusuf on 14/02/2024 02:35:51
Many people have realized that wave propagation and signal processing are easier to analyze in frequency domain, and use Fourier transformation to convert the equation from time domain. But fewer realized that Fourier transformation is just a special case where there's no energy transformation to different forms. The more general transformation to convert from time domain to frequency domain is Laplace transformation. It should be used to analyze light generation and absorption.
Title: Re: Where does quantization of energy of electromagnetic radiation come from?
Post by: hamdani yusuf on 14/02/2024 14:09:09
Maxwell produced his equations before knowing that electric charge is quantized. It was only established after Millikan's oil drop experiment showing the charge of an electron. It's no wonder if the equations don't work well enough for the situations where the effects of that quantization are  no longer negligible.
Title: Re: Where does quantization of energy of electromagnetic radiation come from?
Post by: alancalverd on 14/02/2024 22:08:43
Apropos reply#284, you have expanded my point in #282. There is a lot of energy transfer going on inside the switching crystal, so it's hardly  surprising that what comes out isn't the same as what goes in. If the PhD project was about eliciting the mechanisms, it would have been interesting and valuable, If it was just a phenomenological display of spectral broadening, it should have been covered in the undergraduate syllabus.
Title: Re: Where does quantization of energy of electromagnetic radiation come from?
Post by: hamdani yusuf on 16/02/2024 21:47:59
Let's analyze the most simple case, and see if our models can make sense of it. An electron is made to oscillate at frequency of 1 Hz. The amplitude is 1 meter. How many photons is it radiating every second in average?
What if the amplitude is then reduced to 1 mm?
Is there a minimum amplitude?
Is there a maximum amplitude?
Title: Re: Where does quantization of energy of electromagnetic radiation come from?
Post by: hamdani yusuf on 22/02/2024 06:06:21
Let's analyze the most simple case, and see if our models can make sense of it. An electron is made to oscillate at frequency of 1 Hz. The amplitude is 1 meter. How many photons is it radiating every second in average?
What if the amplitude is then reduced to 1 mm?
Is there a minimum amplitude?
Is there a maximum amplitude?
IMO, the last question would be constrained by speed of light. But the answer to the other questions are less obvious.
It seems like the currently most widely accepted model isn't adequate to give us the satisfactory answers.
Title: Re: Where does quantization of energy of electromagnetic radiation come from?
Post by: hamdani yusuf on 22/02/2024 06:10:59
Where Does Light Come From? (Electrodynamics)
Quote
It's often said that light is an electromagnetic wave, a disturbance in electric and magnetic fields, but what does that mean? How are they made? Let's take a deeper look at electrodynamics and this history behind the discovery to see if we can find an answer.

It shows the timeline for our understanding of light.
Title: Re: Where does quantization of energy of electromagnetic radiation come from?
Post by: alancalverd on 22/02/2024 22:21:50
Is there a maximum amplitude?
Obviously.
Title: Re: Where does quantization of energy of electromagnetic radiation come from?
Post by: hamdani yusuf on 23/02/2024 07:20:39
Is there a maximum amplitude?
Obviously.
Let's analyze the most simple case, and see if our models can make sense of it. An electron is made to oscillate at frequency of 1 Hz. The amplitude is 1 meter. How many photons is it radiating every second in average?
What if the amplitude is then reduced to 1 mm?
Is there a minimum amplitude?
Is there a maximum amplitude?
IMO, the last question would be constrained by speed of light. But the answer to the other questions are less obvious.
It seems like the currently most widely accepted model isn't adequate to give us the satisfactory answers.
Since the electron can't move faster than c, then the maximum distance it can travel in 1 second is around 300 million meter, thus the maximum amplitude is 75 million meter.

Can you help to answer the other questions?
Title: Re: Where does quantization of energy of electromagnetic radiation come from?
Post by: alancalverd on 23/02/2024 08:16:00
You may be about to confuse Planck's "particle in a box" model with a Maxwellian "resonant aerial" model.  Beware!
Title: Re: Where does quantization of energy of electromagnetic radiation come from?
Post by: hamdani yusuf on 24/02/2024 15:14:05
You may be about to confuse Planck's "particle in a box" model with a Maxwellian "resonant aerial" model.  Beware!
I don't assume those models. My assumptions are as follow:
Electric charge is quantized.
An electron is the quantum of negatively charged particle.
It can be isolated.
It can be made to move in certain ways.

Which model do you think is best to describe my example?
Title: Re: Where does quantization of energy of electromagnetic radiation come from?
Post by: hamdani yusuf on 24/02/2024 15:25:18
If two distinct models are required to explain two distinct but related cases, then both models must contain false assumptions which become significant in one of the case.
Title: Re: Where does quantization of energy of electromagnetic radiation come from?
Post by: alancalverd on 24/02/2024 23:44:13
Which model do you think is best to describe my example?
Since your example is of an oscillating current at 1 Hz, Maxwell will do the job very well. It is indeed used by those who design VLF antennae for undersea communication.

The cases are distinct and unrelated.
Title: Re: Where does quantization of energy of electromagnetic radiation come from?
Post by: hamdani yusuf on 25/02/2024 04:24:37
This video contains historical context for Planck's law, which is mostly dismissed in textbooks.
Quote
Story of how Planck discovered the blackbody radiation formula and why he introduced energy quantization as a math trick

Errata:
08:10​ instead of Pringscheim should be Pringsheim, thanks to @petermarksteiner7754​ for notifying this
14:40​ after the integration there is an extra minus sign that should not be there, thanks @escandestone6001​ for notifying this
20:22​ second equation should be ε/(kT)=log(1+ε/U), thanks to @Galileosays​ for notifying this
23:52​ "gasses" should be "gases," thanks to @Robert-skibelo​ for notifying this
Title: Re: Where does quantization of energy of electromagnetic radiation come from?
Post by: hamdani yusuf on 25/02/2024 04:30:59
The cases are distinct and unrelated.
Just multiply the frequency by 1 quadrillion times, and you get ultraviolet light. That's a relationship, no matter how hard you try to deny it.
Title: Re: Where does quantization of energy of electromagnetic radiation come from?
Post by: alancalverd on 26/02/2024 15:14:48
Ultraviolet maybe, but continuous emission at any frequency you choose.
Title: Re: Where does quantization of energy of electromagnetic radiation come from?
Post by: alancalverd on 26/02/2024 15:16:13
which is mostly dismissed in textbooks.

Not in mine! But that was 60 years ago. Maybe they don't teach physics anymore.
Title: Re: Where does quantization of energy of electromagnetic radiation come from?
Post by: hamdani yusuf on 03/03/2024 04:32:55
In Planck's formula for radiation from black bodies, Boltzmann's constant somehow appears.
(https://upload.wikimedia.org/wikipedia/commons/thumb/6/62/Ideal_gas_law_relationships.svg/400px-Ideal_gas_law_relationships.svg.png)

https://en.m.wikipedia.org/wiki/Boltzmann_constant
Quote
Macroscopically, the ideal gas law states that, for an ideal gas, the product of pressure p and volume V is proportional to the product of amount of substance n and absolute temperature T:
 pV=nRT,
where R is the molar gas constant (8.31446261815324 J⋅K−1⋅mol−1).[3] Introducing the Boltzmann constant as the gas constant per molecule[4] k = R/NA transforms the ideal gas law into an alternative form:
 pV=NkT,
where N is the number of molecules of gas.

It implies the quantization of oscillating electric charges producing the radiation,  which is related to the definition of temperature itself.
Title: Re: Where does quantization of energy of electromagnetic radiation come from?
Post by: alancalverd on 07/03/2024 11:38:29
No such implication. kB derives from classical statistical mechanics.
Title: Re: Where does quantization of energy of electromagnetic radiation come from?
Post by: hamdani yusuf on 07/03/2024 12:26:27
kB derives from classical statistical mechanics.
Do you meant it's no longer needed?
Title: Re: Where does quantization of energy of electromagnetic radiation come from?
Post by: alancalverd on 07/03/2024 19:29:52
Of course it's needed. How else can you describe the Boltzmann distribution, rationalise the gas laws, or do thermodynamics?

Quantum mechanics doesn't displace classical mechanics, any more than relativity displaces newtonian physics. The test of both is that they degenerate to the classical formula for large or slow systems.
Title: Re: Where does quantization of energy of electromagnetic radiation come from?
Post by: hamdani yusuf on 08/03/2024 05:14:44
Of course it's needed. How else can you describe the Boltzmann distribution, rationalise the gas laws, or do thermodynamics?

Quantum mechanics doesn't displace classical mechanics, any more than relativity displaces newtonian physics. The test of both is that they degenerate to the classical formula for large or slow systems.
How does Boltzmann's constant relate to black body radiation?
Title: Re: Where does quantization of energy of electromagnetic radiation come from?
Post by: hamdani yusuf on 08/03/2024 07:47:01
I asked Gemini. What is the difference between planck's formula for black body radiation and wien's approximation?
Quote
Planck's formula and Wien's approximation both attempt to describe the spectrum of black body radiation, the characteristic energy distribution emitted by an ideal perfect absorber at a given temperature. However, they differ in their accuracy and underlying assumptions.
...
In conclusion, Planck's formula is the more general and accurate description of black body radiation, thanks to its inclusion of energy quantization. Wien's approximation provides a useful simplification for shorter wavelengths but is not sufficient for the entire spectrum.

Can you write the equations side by side?
Quote
Planck's Formula   B(ν, T) = (2 * h * ν^3) / (c^2 * (e^(h * ν / (k_B * T)) - 1))
Wien's Approximation   B(ν, T) = (2 * h * ν^3) / (c^2 * e^(h * ν / (k_B * T)))

It seems like planck only adds -1 term to wien's formula.
Quote
You're right, at first glance, the equations might seem very similar, with Planck's formula just adding a -1 term to Wien's approximation. However, that seemingly small addition has a profound impact:

The -1 term accounts for quantization: This term incorporates the revolutionary idea that energy can only be emitted or absorbed in discrete packets. This concept of quantized energy levels was a major breakthrough in physics and went beyond the classical framework used in Wien's approximation.

Title: Re: Where does quantization of energy of electromagnetic radiation come from?
Post by: alancalverd on 08/03/2024 08:42:26
Just shows why you shouldn't use ChatGPT as a source of scientific information. It displays all the insight and understanding of a politician.
Title: Re: Where does quantization of energy of electromagnetic radiation come from?
Post by: hamdani yusuf on 08/03/2024 11:05:09
Just shows why you shouldn't use ChatGPT as a source of scientific information. It displays all the insight and understanding of a politician.
It reflects the quality of training data sources. It seems like it hasn't been equipped with the tools to filter out incorrect statements.
Title: Re: Where does quantization of energy of electromagnetic radiation come from?
Post by: alancalverd on 09/03/2024 12:47:32
My point exactly!
Title: Re: Where does quantization of energy of electromagnetic radiation come from?
Post by: hamdani yusuf on 09/03/2024 22:37:09
Just shows why you shouldn't use ChatGPT as a source of scientific information. It displays all the insight and understanding of a politician.
It reflects the quality of training data sources. It seems like it hasn't been equipped with the tools to filter out incorrect statements.
When the AI models still depend on their human supervisors to select which data sources are more accurate and reliable than the others, then the quality of those supervisors will determine the quality of responses that they'll produce. Leaving their current mistakes uncorrected will make it harder and more expensive to correct them in the future.
Title: Re: Where does quantization of energy of electromagnetic radiation come from?
Post by: alancalverd on 10/03/2024 00:38:56
So the only trustworthy answers come from human judgement, and may become more trustworthy as humans test their hypotheses by experiment (science) but anything from an AI source may be infected by untruth, and any such infection is potentially malignant because the whole point of AI is that it can collect inputs (including its own output) and synthesise outputs quicker than a human. Which puts AI on a par with religion.
Title: Re: Where does quantization of energy of electromagnetic radiation come from?
Post by: hamdani yusuf on 10/03/2024 08:55:12
anything from an AI source may be infected by untruth
Human sources are not immune from falsehood either. The probabilities for their occurrence can only be reduced by more check and recheck, compared to observations.
Title: Re: Where does quantization of energy of electromagnetic radiation come from?
Post by: alancalverd on 11/03/2024 22:55:09
But humans tend to conduct experiments or simply ask "does that seem reasonable?"   AI just goes on regurgitating any old rubbish because it doesn't care about the consequences of its output.
Title: Re: Where does quantization of energy of electromagnetic radiation come from?
Post by: hamdani yusuf on 12/03/2024 11:59:49
But humans tend to conduct experiments or simply ask "does that seem reasonable?"   AI just goes on regurgitating any old rubbish because it doesn't care about the consequences of its output.
But the developers do care about the accuracy of their AI models. It determines whether or not they can get their return of investment.
Title: Re: Where does quantization of energy of electromagnetic radiation come from?
Post by: alancalverd on 12/03/2024 16:50:53
The model may be 100% accurate and quote Donald Trump's authoritative speech to the letter, but I wouldn't want anyone to inject themselves with hydrochloroquinone.
Title: Re: Where does quantization of energy of electromagnetic radiation come from?
Post by: hamdani yusuf on 15/03/2024 11:56:13
The model may be 100% accurate and quote Donald Trump's authoritative speech to the letter, but I wouldn't want anyone to inject themselves with hydrochloroquinone.
New AI models will be able to filter out misinformation, based on previously accumulated information, and comparison with various other information sources, including their own sensors.
Title: Re: Where does quantization of energy of electromagnetic radiation come from?
Post by: alancalverd on 16/03/2024 16:43:25
If all the previously accumulated information is the Bible, AI will reject any scientific statement as unreliable and the world will regress to the intellectual level of the Republican Party.

Science is all about challenging consensus, not nurturing it.
Title: Re: Where does quantization of energy of electromagnetic radiation come from?
Post by: hamdani yusuf on 17/03/2024 09:15:15
If all the previously accumulated information is the Bible, AI will reject any scientific statement as unreliable and the world will regress to the intellectual level of the Republican Party.

That's a big if. And fortunately it's not true.
Title: Re: Where does quantization of energy of electromagnetic radiation come from?
Post by: alancalverd on 17/03/2024 10:16:40
It was once. Similar problems have been found where all the permitted truth was all in Mao's Little Red Book, or Lysenko's state-approved botany papers.

Problem with the internet as a source is that there is only one right answer to scientific question, but an infinite number of wrong ones, and the internet grants allows them equal weight.
Title: Re: Where does quantization of energy of electromagnetic radiation come from?
Post by: hamdani yusuf on 17/03/2024 16:12:30
It was once. Similar problems have been found where all the permitted truth was all in Mao's Little Red Book, or Lysenko's state-approved botany papers.

Problem with the internet as a source is that there is only one right answer to scientific question, but an infinite number of wrong ones, and the internet grants allows them equal weight.
It would be harder to censor worldwide openly accessible information sources. Especially with VPN and satellite connection.

Internet doesn't give equal weight for different sources. Traditional search engines like Google prioritize most referred sources.
Title: Re: Where does quantization of energy of electromagnetic radiation come from?
Post by: alancalverd on 18/03/2024 09:04:17
Traditional search engines like Google prioritize most referred sources.
...like The Bible, Koran, Little Red Book, Shakespeare, Alice in Wonderland, and other frequently-cited sources of utter nonsense.

At best, consensus represents the opinion of the average man. Advances in understanding are made by exceptional people questioning the consensus. Out of 8,000,000,000 people, no more than 18 are deemed worthy of Nobel prizes each year, and if we discount those that don't involve increasing knowledge, we are down to 9. 
Title: Re: Where does quantization of energy of electromagnetic radiation come from?
Post by: hamdani yusuf on 18/03/2024 15:30:09
At best, consensus represents the opinion of the average man.
That's what democracies depend on. Democratic societies whose constituents are mostly incompetent are less likely to survive. Usually, least competent members of the societies would die out earlier. It would tip the average value upward.
Title: Re: Where does quantization of energy of electromagnetic radiation come from?
Post by: hamdani yusuf on 18/03/2024 15:34:14
Advances in understanding are made by exceptional people questioning the consensus.
There are some types of research gaps.
Quote
In this video, I dive deep into the landscape of academic research. I share insights from my own journey, focusing on the discovery and exploration of research gaps. These gaps, often seen as voids in our understanding, are in fact golden opportunities for innovation and progress. I discuss four distinct types of research gaps that I've encountered, including the personal challenges and triumphs I've experienced in bridging them.
Title: Re: Where does quantization of energy of electromagnetic radiation come from?
Post by: hamdani yusuf on 19/03/2024 08:08:33
At best, consensus represents the opinion of the average man.
Or average researchers.

How To Use Consensus AI - Don?t Get Left Behind

Quote
Embark on a journey to redefine your academic search with Consensus AI, the cutting-edge tool transforming the landscape of research.
TIMESTAMPS
00:00 How to use Consensus
00:48 How to Use AI Summary
02:43 How to use Consensus Meter
03:23 How to use Consensus Co Pilot
04:47 How to Filter with Consensus
06:40 Digging Deeper into Consensus
08:41 Consensus on the GPT Store
09:38 Wrapping Up
Title: Re: Where does quantization of energy of electromagnetic radiation come from?
Post by: alancalverd on 19/03/2024 10:51:34
Democratic societies whose constituents are mostly incompetent are less likely to survive.
It is a characteristic of most human societies that we protect and nurture the incompetent and superstitious, and even insist that they have a right to breed and/or disseminate their superstitions. Hence
Quote
Usually, least competent members of the societies would die out earlier.
is no longer a "given"
and
Quote
It would tip the average value upward.
is clearly not the case in, for example, the USA or anywhere that religion or party/tribal loyalty influences the democratic process.
Title: Re: Where does quantization of energy of electromagnetic radiation come from?
Post by: hamdani yusuf on 19/03/2024 22:35:55
is clearly not the case in, for example, the USA or anywhere that religion or party/tribal loyalty influences the democratic process.
There might be some exceptions in short terms, for whatever reason. But in the long term the trend is usually maintained.
Title: Re: Where does quantization of energy of electromagnetic radiation come from?
Post by: hamdani yusuf on 02/05/2024 09:12:01
In atoms, only certain orbitals are permitted (quantization)
- To calculate these orbitals, you need to solve the wave equation for the electron.
- Some of these orbitals are spherical, but others look like a cluster of balloons assembled by a clown. How do you calculate the radius and centripetal motion for these?


(https://upload.wikimedia.org/wikipedia/commons/thumb/5/58/Neon_orbitals.png/1280px-Neon_orbitals.png)
The shapes of the first five atomic orbitals are: 1s, 2s, 2px, 2py, and 2pz. The two colors show the phase or sign of the wave function in each region. Each picture is domain coloring of a ψ(x, y, z) function which depends on the coordinates of one electron.

(https://upload.wikimedia.org/wikipedia/commons/thumb/e/e7/Hydrogen_Density_Plots.png/1280px-Hydrogen_Density_Plots.png)
Atomic orbitals of the electron in a hydrogen atom at different energy levels. The probability of finding the electron is given by the color, as shown in the key at upper right.

https://en.m.wikipedia.org/wiki/Atomic_orbital


These atomic orbitals may not represent position of electrons around the nucleus. For me, it looks more like radiation patterns around antenna.
(https://www.antenna-theory.com/antennas/norm3D1lam.jpg)
Figure 4. Normalized 3d radiation pattern for the 1-wavelength dipole antenna.

(https://www.antenna-theory.com/antennas/norm3D1p5.jpg)
Figure 5. Normalized 3d radiation pattern for the 1.5-wavelength dipole antenna.
https://www.antenna-theory.com/antennas/dipole.php


(https://media.cheggcdn.com/media/ae7/ae756df1-ae0b-49a6-a4c3-39cc0569a3c0/phpPrnrJB)
From chegg.com

Spatially, the patterns extend to infinity, although with quickly dropped magnitude for further distances. While dark regions within finite distances might be interpreted as products of destructive interference.

Unlike in a dipole antenna where the movement of electrons is confined in one dimension, the electrons around atomic nucleus can move in three dimensions.
Title: Re: Where does quantization of energy of electromagnetic radiation come from?
Post by: hamdani yusuf on 02/05/2024 10:17:03
Spherical Harmonics
Quote
We describe the possible fundamental vibrations on a sphere in three dimensions by counting, mirroring and rotating nodal lines.
This video ist part of the online course www.quantumreflections.net dealing with quantum physics, produced by the institute for physics education research, M?nster university (Germany)
Title: Re: Where does quantization of energy of electromagnetic radiation come from?
Post by: hamdani yusuf on 02/05/2024 15:38:59
Spatially, the patterns extend to infinity, although with quickly dropped magnitude for further distances. While dark regions within finite distances might be interpreted as products of destructive interference.

Unlike in a dipole antenna where the movement of electrons is confined in one dimension, the electrons around atomic nucleus can move in three dimensions.
Based on the radiation patterns that we can measure for various shapes of antenna where we can control the motion of the electrons, we can calculate backwards to infer how electrons should move around the nucleus to produce radiation patterns resembling known atomic orbitals. It will require adequately accurate and precise electrodynamic model.
Title: Re: Where does quantization of energy of electromagnetic radiation come from?
Post by: hamdani yusuf on 02/05/2024 16:04:51
It's worth noting that Schrodinger himself didn't like Max Born's interpretation of his wave equation for atomic orbitals (after being squared) as probability density of finding electrons in particular region.  One of the objections to the implications of that interpretation is the formation of two or more separated regions in space where electrons can be found, but not between them. It's as if electrons can magically jump from one region with high probability to another regions without having to travel the space between them.
Title: Re: Where does quantization of energy of electromagnetic radiation come from?
Post by: alancalverd on 03/05/2024 09:04:25
Based on the radiation patterns that we can measure for various shapes of antenna where we can control the motion of the electrons, we can calculate backwards to infer how electrons should move around the nucleus to produce radiation patterns resembling known atomic orbitals. It will require adequately accurate and precise electrodynamic model.
It is precisely the absence of electromagnetic emission from atoms in equilibrium that put an end to the Bohr atom.
It's as if electrons can magically jump from one region with high probability to another regions without having to travel the space between them.
There's no implication that they magically jump. If you start with a classical model, you probably won't derive a quantum result, but if you start with a quantum and probabilistic model, you can usefully predict molecular orbitals that correspond with ball-and-stick models, crystallographic measurements, and observed spectra.
Title: Re: Where does quantization of energy of electromagnetic radiation come from?
Post by: hamdani yusuf on 03/05/2024 13:05:54
It is precisely the absence of electromagnetic emission from atoms in equilibrium that put an end to the Bohr atom.
No. It's why Bohr proposed his model, against Rutherford's planetary model.
Title: Re: Where does quantization of energy of electromagnetic radiation come from?
Post by: hamdani yusuf on 03/05/2024 13:18:28
There's no implication that they magically jump.
You can say that they casually leap or tunnel. But what matters is what it means. It suggests that electrons can disappear from their previous positions and reappear in new position without traveling through the space in between.
Title: Re: Where does quantization of energy of electromagnetic radiation come from?
Post by: alancalverd on 03/05/2024 17:48:56
You can say that they casually leap or tunnel.
No. Tunneling is an entirely different phenomenon.

It suggests that electrons can disappear from their previous positions and reappear in new position without traveling through the space in between.

No. Probability does not imply that anything moves. The probability of encountering a person with a black face in the street in Kinshasa is close to 100%, in Manchester, about 5%. There is no implication of instantaneous transport between the two cities.
Title: Re: Where does quantization of energy of electromagnetic radiation come from?
Post by: alancalverd on 03/05/2024 17:51:31
No. It's why Bohr proposed his model, against Rutherford's planetary model.
Bohr's electrons orbit, That's the problem.
Title: Re: Where does quantization of energy of electromagnetic radiation come from?
Post by: hamdani yusuf on 04/05/2024 00:14:23
No. It's why Bohr proposed his model, against Rutherford's planetary model.
Bohr's electrons orbit, That's the problem.
He postulated that some orbits are stable, hence electrons occupying them don't radiate energy.
Title: Re: Where does quantization of energy of electromagnetic radiation come from?
Post by: hamdani yusuf on 04/05/2024 00:16:35
No. Probability does not imply that anything moves.
How can electrons have momentum if they don't move?
Title: Re: Where does quantization of energy of electromagnetic radiation come from?
Post by: hamdani yusuf on 04/05/2024 00:17:50
No. Tunneling is an entirely different phenomenon.
Are you fine with the word leap?
Title: Re: Where does quantization of energy of electromagnetic radiation come from?
Post by: alancalverd on 04/05/2024 11:15:18
He postulated that some orbits are stable, hence electrons occupying them don't radiate energy.
Orbiting equals acceleration. Accelerating charges emit electromagnetic radiation. So the classical model is wrong.
Title: Re: Where does quantization of energy of electromagnetic radiation come from?
Post by: alancalverd on 04/05/2024 11:16:25
No. Tunneling is an entirely different phenomenon.
Are you fine with the word leap?
Different phenomena.
Title: Re: Where does quantization of energy of electromagnetic radiation come from?
Post by: hamdani yusuf on 08/05/2024 13:54:36
No. Tunneling is an entirely different phenomenon.
Are you fine with the word leap?
Different phenomena.
Which phenomena are you referring to?
Title: Re: Where does quantization of energy of electromagnetic radiation come from?
Post by: hamdani yusuf on 08/05/2024 13:56:44
Accelerating charges emit electromagnetic radiation.
Not necessarily. Circulating current in a ring superconductor can stay for years with no perceivable decay.
Title: Re: Where does quantization of energy of electromagnetic radiation come from?
Post by: hamdani yusuf on 08/05/2024 13:58:23
He postulated that some orbits are stable, hence electrons occupying them don't radiate energy.
Orbiting equals acceleration. Accelerating charges emit electromagnetic radiation. So the classical model is wrong.
Bohr's model is not usually considered classical.
Title: Re: Where does quantization of energy of electromagnetic radiation come from?
Post by: alancalverd on 08/05/2024 17:48:54
Classical electrodynamics still applies. An accelerating charge emits em radiation, by observation. Therefore any model that involves indefinitely accelerating charges that do not emit radiation, is wrong.
Title: Re: Where does quantization of energy of electromagnetic radiation come from?
Post by: alancalverd on 08/05/2024 17:49:29
Which phenomena are you referring to?
Jumping and tunneling.
Title: Re: Where does quantization of energy of electromagnetic radiation come from?
Post by: alancalverd on 08/05/2024 17:50:32
Accelerating charges emit electromagnetic radiation.
Not necessarily. Circulating current in a ring superconductor can stay for years with no perceivable decay.
Which is why you need quantum mechanics to explain superconductivity.